NVLE FELINE

Ace your homework & exams now with Quizwiz!

b. Parenteral terbutaline with supplemental oxygen The history, physical exam, and radiographs are consistent with a diagnosis of feline asthma. The radiographic findings in this case are a diffuse bronchial pattern consisting of airway thickening appearing as so called "tracks and doughnuts." Terbutaline is a beta-2 agonist that allows bronchial smooth muscle relaxation and along with oxygen, are two important aspects of the emergency management of a cat in acute respiratory distress from asthma. In reality, you probably should have administered these therapies prior to radiographs as the stress of handling a cat in acute respiratory distress can be dangerous. Along those lines, restraining this cat does not have either of those problems at this time

A 10-year-old female spayed cat presents to your emergency clinic with an acute onset of coughing and dyspnea. On physical exam, you detect open mouth breathing, a marked abdominal effort, and a respiratory rate of 54 breaths per minute. Her temperature is 102.7F (39.3C), and her chest radiographs appear as below. What is the most appropriate next step for this cat? a. Thoracocentesis to remove air from the pleural space b. Parenteral terbutaline with supplemental oxygen c. Thoracocentesis to remove fluid from the pleural space d. Parenteral ampicillin and enrofloxacin with supplemental oxygen e. CBC, Chemistry panel, urinarlysis and a heartworm test

b. Humulin-R (regular insulin) This cat is in a state of diabetic ketoacidosis. Due to this status, the recommended insulin type is Humulin-R, or regular insulin. This is a shorter acting insulin which will help to get the ketouria to resolve more efficiently. After the ketosis has resolved and the cat is hydrated, eating, drinking, and electrolytes are stable, the cat can be switched to a longer acting insulin. Glargine or PZI are the insulin types of choice for long term control in felines after the ketoacidosis has been corrected, but other insulin types can also be used.

A 10-year-old female spayed domestic short hair named Gypsy presents with a history of increased drinking and urinating for 2 weeks and weight loss. She has not eaten in 2 days. Bloodwork shows blood glucose 458mg/dL and her urine shows 3+ glucose, 2+ ketones. Which type of insulin would be recommended for this patient at this time? a. Glargine (Lantus) insulin b. Humulin-R (regular insulin) c. Humulin-N (NPH) d. Vetsulin e. PZI insulin

a. Blood pressure Cats with hyperthyroidism are likely to develop hypertension. If this is severe enough (>180-200 mmHg systolic), they can be at risk for acute retinal detachment or hemorrhage resulting in blindness. Prompt resolution of the hypertension is critical to prevent further damage to the eye and other organs.

A 10-year-old male castrated cat that you have previously diagnosed with hyperthyroidism presents to you for acute onset of blindness. You perform an ophthalmic exam and note retinal hemorrhage. What diagnostic test should you perform first? a. Blood pressure b. Total T4 levels c. Serum BUN and creatinine d. Free T4 levels by equilibrium dialysis e. Coagulation times

e. Hyperthyroidism

A 10-year-old spayed female cat is being evaluated because of weight loss, polyphagia, polyuria, polydipsia, and restlessness. Cardiac arrhythmias are noted on auscultation. Which of the following is the most likely diagnosis? a. Chronic pancreatic insufficiency b. Chronic renal disease c. Congenital cardiomyopathy d. Diabetes mellitus e. Hyperthyroidism

d. Surgery The radiographs show the colon in the thoracic cavity. This finding is consistent with a diaphragmatic hernia. The best course of treatment is surgery to return the abdominal organs back into their appropriate place and close the defect in the diaphragm. Note that the cat has an increased respiratory rate, due to the colon being the chest.

A 10-yearold cat presents with a history of chronic constipation and hasn't had a bowel movement in several days. The owner says the cat is uncomfortable and breathing heavy when she strains to defecate. T=102.8F (39.3C), P=220, R= 75. You take radiographs (see image). Which course of treatment is most appropriate for this cat? a. Manual deobstipation b. Lactulose and Cisapride c. Thorachocentesis d. Surgery

c. Free T4 by equilibrium dialysis An elevated free T4 by equilibrium dialysis is a way to support the diagnosis of occult hyperthyroidism. In a small subset of cats, the free T4 is elevated due to non-thyroidal illness. Therefore, hyperthyroidism should not be diagnosed on a free T4 measurement alone. Most older cats have a thyroid level that is in the lower end of the normal range. The T4 in this cat is in the high end of the normal range. In combination with the exam findings and clinical signs, this raises suspicion for hyperthyroidism. Many hyperthyroid cats have hypertension so this is a test that should be checked. However, many older cats also have hypertension that do not have hyperthyroidism so this would not confirm the diagnosis. The same applies with appearance of the heart on radiographs. Many hyperthyroid cats will have thickening of the ventricles and an enlarged heart on radiographs, but this radio

A 12-year-old cat with an unkept haircoat and palpable thyroid slip presents with a history of weight loss. Appetite has been normal to increased. Bloodwork is unremarkable and total T4 level is 3.8 ug/dL (normal 0.8-4.0). ALthough the value is within the reference range, you still suspect hyperthyroidism. Which other test could best support the diagnosis? a. Chest radiographs showing a valentine shaped heart, or evidence of HCM b. Ultrasound of the thyroid gland c. Free T4 by equilibrium dialysis d. Serum thyroglobulin antibodies e. Blood pressure if hypertension is present

e. Linear foreign body The appearance of plicated bowel in a cat that is not eating and is vomiting is typical of a linear foreign body. If a string or linear object is swallowed, it can anchor in the intestinal tract cranially and the intestine can then start to bunch up like an accordion. It is very important to do a good oral exam to ensure that a string is not anchored under the tongue. This appearance on a radiograph warrants exploratory surgery,

A 2-year-old FS domestic short hair presents with a history of anorexia and vomiting for the last 48 hours. She is dehydrated and when you palpate her abdomen she seems uncomfortable. Bloodwork shows HCT 55% (30-45%), albumin 4.5g/dL (2.8-3.9g/dL), sodium 132 mEq/L (146-156 mEq/L), otherwise unremarkable. Temeprature is 102.5F. You perform abdominal radiographs and see that the intestines appear to be bunched up in accordion-like fashion. The intestines are gas filled and moderately dilated. This plicated appearance is most typical of: a. Mesenteric volvulus b. Ileus c. Free gas in the abdomen from a perforation d. Normal peristalsis e. Linear foreign body f. Aerophagia

c. Hypoallergenic diet and prednisolone for inflammatory bowel disease Lymphocytic plasmacytic enteritis is consistent with inflammatory bowel disease. The main treatments of this disease include controlling the underlying cause for the disease, controlling inflammation, and controlling bacterial overgrowth when needed, Steroids and hypoallergenic diet are the mainstay treatments for this disease. Prednisolone and budesonide are the two corticosteroids most often used. Metronidazole can also aid in treatment. Cisplastin is a chemotherapeutic agent that is contraindicated for the use in cats for any disease. Fortiflora is a probiotic that could possibly help with clinical signs of IBD, however, is not necessarily a treatment for the disease itself Metoclopramide could potentially help clinical vomiting from underlying IBD, but is a pro-motility agent that could worsen diarrhea and is not a treatment for IBD WHile

A 12-year-old indoor only Devon Rex presents with a 2 month history of weight loss, decreased appetite, and diarrhea with occasional vomiting. Bloodwork is unremarkable. The closest referral for ultrasound or endoscopy is 2 hours away and the owner is unable to drive for further diagnostics. You perform an abdominal exploratory and take biopsies of the stomach and intestine. The histopathology results show lymphocytic plasmacytic enteritis with a moderate number of eosinophils. Which of the following is the best recommendation for this cat? a. Prednisolone and cisplastin for lymphoma b. Fenbendazole to treat the likely occult Giardia infection c. Hypoallergenic diet and prednisolone for inflammatory bowel disease d. Metocloparamide to assist with GI motility and decrease nausea e. Fortiflora to restore normal intestinal flora

A 12-year-old male neutered domestic short hair cat presents for lethargy and inappetence. It has a history of diabetes mellitus that the owner has been treating with 2 units of PZI insulin subcutaneously twice daily. Your initial exam and lab work show the cat is in diabetic ketoacidosis. How should you treat the cat? Animals in DKA should be rehydrated with IV fluids and aggressively treated with frequent doses (or a CRI) of short-acting insulin (regular insulin) to obtain glycemic control before making any alterations to its normal insulin regimen at home. Careful blood glucose monitoring should be instituted as well to make sure the animal does not become hypoglycemic. An underlying cause for its sudden poor glycemic control should also be investigated. Commonly, underlying conditions such as pancreatitis, urinary tract infection, or other underlying infections are responsible for disrupting a diabetic's glycemi

A 12-year-old male neutered domestic short hair cat presents for lethargy and inappetence. It has a history of diabetes mellitus that the owner has been treating with 2 units of PZI insulin subcutaneously twice daily. Your initial exam and lab work show the cat is in diabetic ketoacidosis. How should you treat the cat? a. Treat with frequent doses or constant rate infusion (CRI) of regular insulin initially to get the cat underglycemic control b. Send the cat home on an appetite stimulant and increased dose of PZI insulin c. Switch the PZI insulin to a longer acting insulin, such as ultralante, keeping the dosing the same d. Switch PZI insulin to lente insulin and increase the dosing e. Switch the PZI insulin to lente insulin but keep the same doing

c. Magnetic resonance imaging (MRI) of the head This cat has the signs and symptoms of acromegaly. Acromegaly is caused by excessive growth hormone release from the pars distalis from a tumor in the pituitary gland. Excessive growth hormone causes a defect in the insulin receptors on target cells causing insulin resistant diabetes mellitus. The enlarged head, paws, abdomen, and weight gain despite uncontrolled diabetes is due to the anabolic effects of the growth hormone. Treatment for this condition include radiation therapy to the pituitary tumor, high doses of insulin to try and control the diabetes, and somatostatin analogs (octreotide) to try and inhibit the release of growth hormone from the tumor. Surgical excision has been used as a form of treatment in people with pituitary tumors, but this has only been rarely reported in cats.

A 12-year-old male neutered domestic short hair cat presents for ongoing evaluation of diabetes mellitus. The cat was diagnosed 6 months ago and has continued to be markedly polyuric, polydipsic, polyphagic, and has been gaining weight. The cat is currently receiving 10 units of glargine insulin every 12 hours. On physical exam, the cat weighs 15 pounds (6.8kg) and has an enlarged head, abdomen, and paws. What imaging modality would be most appropriate to try and prove what you suspect is causing the uncontrolled diabetes and weight gain in this cat? a. Radiographs of the thorax b. Ultrasound of the neck c. Magnetic resonance imaging (MRI) of the head d. Ultrasound of the abdomen e. Computed tomography (CT scan) of the abdomen

a. Hypokalemic myopathy The posture is classic for hypokalemia and other neuromyopathies (e.g., myasthenia gravis, organophosphate intoxication) in cats. Hyperkalemic periodic paralysis has not been reported in cats.

A 12-year-old spayed female cat is presented with a history of weakness for the past 2 days. She has neck ventroflexion and a stiff, stilted gait. Which one of the following conditions is in the top of the differential list? a. Hypokalemic myopathy b. Hyperkalemic periodic paralysis c. Cervical vertebral malformation d. Caudal occipital malformation e. Bilateral otitis media/interna

a. The cat has primary hyperparathyroidism and surgery is recommended. The symptoms, physical exam findings, and blood work results are most consistent with primary hyperparathyroidism, which is most commonly caused by a parathyroid gland tumor. The elevated calcium causes the signs of PU/PD. Cats will commonly not show significant clinical signs and may be diagnosed incidentally on routine blood work evaluation. This cat's clinical signs of PU/PD and slight lethargy are non-specific, but classic for primary hyperparathyroidism along with the blood work shown in the question, which depicts borderline or mild azotemia, high total and ionized calcium levels, and a low phosphorus.

A 13-year-old female spayed domestic short hair cat presents for a geriatric screening. The owner reports that the cat is polyuric, polydipsic, and slightly lethargic recently. Physical exam reveals a nodule in the region of the thyroid gland and a thin body condition. Blood work is run with the following findings (normal ranges are within parentheses): T4 2.7ug/dL (2.3-4.7 ug/dL), blood urea nitrogen 34 mg/dL (10-32 mg/dL), creatinine 2.1 mg/dL (0.5-2.2 mg/dL), total calcium 16 mg/dL (8-11 mg/dL), phosphorus 2.5 mg/dL (3.5-8.1 mg/dL). An ionized calcium is 1.8 mmol/L (1.12-1.32 mmol/L). A urinalysis shows a urine specific gravity of 1.010 and numerous calcium oxalate crystals. An ultrasound of the neck confirms a nodule within the region of the thyroid. What do you tell the owner? a. The cat has primary hyperparathyroidism and surgery is recommended. b. The cat is most likely hyperthyroid and methimazole is recommended. The T4 is within the normal range because of sick euthyroid syndrome c. The cat has lymphoma causing the changes in blood work and staging tests are recommended d. Cats has a urinary tract infection and antibiotics are recommended. e. The cat is in renal failure and should be hospitalized on aggressive intravenous fluids

d. The cat has primary hyperparathyroidism and surgery is recommended The symptoms, physical exam findings, and blood work results are most consistent with primary hyperparathyroidism, which is most commonly caused by a parathyroid gland tumor. The elevated calcium causes the signs of PU/PD. Cats will commonly not show significant clinical signs and may be diagnosed incidentally on routine blood work evaluation. This cat's clinical signs of PU/PD and slight lethargy are non-specific, but classic for primary hyperparathyroidism along with the blood work shown in the question, which depicts borderline or mild azotemia, high total and ionized calcium levels, and a low phosphorus

A 13-year-old female spayed domestic short hair cat presnts for a geriatric screening. The owner reports that the cat is polyuric, polydipsic, and slightly lethargic recently. Physical exam reveals a nodule in the region of the thyroid gland and a thin body condition. Blood work is run with the following findings (normal ranges are within parenthese): T4 2.7 ug/dL (2.3-4.7 ug/dL), blood urea nitrogen 34mg/dL (10-32 mg/dL), creatinine 2.1ng/dL (0.5-2.2mg/dL), total calcium 16mg/dL (8-11 mg/dL), phosphorus 2.5mg/dL (3.5-8.1mg/dL). An ionized calcium is 1.8 mmol/L (1.12-1.32 mmol/L). A urinalysis shows a urine specic gravity of 1.010 and numerous calcium oxalate crystals. An ultrasound of the neck confirms a nodule within the region of the thyroid. What do you tell the owner? a. The cat has lymphoma causing the changes in bloodwork and staging tests are recommended b. The cat is in renal failure and should be hospitalized on aggressive intravenous fluids c. The cat has a urinary tract infection and antibiotics are recommended d. The cat has primary hyperparathyroidism and surgery is recommended e. The cat is most likely hyperthyroid and methimazole is recommended. The T4 is within the normal range because of sick euthyroid syndrome

e. Serum T4 This cat is most likely hyperthyroid. Polyphagia, weight loss, and hyperactivity are classical clinical signs in a cat with hyperthyroidism. Other signs might include a dull haircoat, PU/PD, vomiting, diarrhea, and aggression. Lethargy, anorexia, and weakness are uncommon, but can occur in a condition apathetic hyperthyroidism

A 13-year-old male castrated domestic long hair presents for polyphagia, weight loss, and vomiting. The owner notes that the cat appears restless, more active, and more aggressive than before. Which test would diagnose the most likely cause the these signs? a. Liver panel b. Gastroduodenoscopy and biopsy c. Abdominal ultraosund d. Renal panel and urinalysis e. Serum T4

a. Chemistry panel A chemistry panel should be run first to get a baseline assessment of renal function and liver function. Occult renal insufficiency is often masked by the hemodynamics of hyperthyroidism. Hyperthyroidism often causes increased renal perfusion and increased GFR. In these cats, when hyperthyroidism is treated, the renal insufficiency is unmasked and they become azotemic. For this reason, most clinicians start treatment with methimazole, and recheck renal function when the T4 is normal. If there is no evidence of underlying renal disease, a permanent treatment for hyperthyroidism can then be recommended, such as I-131 therapy. The liver enzymes and liver function tests on a chemistry panel should also be assessed since medical management of hyperthyroidism with methimazole can be hepatotoxic. Hyperthyroidism itself can cause increases in liver enzymes, so it is ideal to have baseline liver enzymes b

A 14-year-old male castrated domestic long hair was diagnosed with hyperthyroidism in your clinic. What additional test should be run before starting treatment for hyperthyroidism? a. Chemistry panel b. Bile acid c. Thoracic radiographs d. CBC

d. Left cerebrum Remember that conscious proprioception and visual perception are affected by lesions in the CONTRALATERAL cerebral hemisphere. Circling direction is generally TOWARD the side of the lesion. Neurologic lesion localization can be challenging. Here are some summary points that may help in this case. What signs make the lesion cerebral? - Standing in corners (head pressing) - Wandering in circles to the left (toward side of lesion) - Contralateral visual loss with normal plapebral and pupillary right reflexes What is the lesion NOT in the brainstem? - Visual pathways do not course through the brainstem, so the visual loss does not fit Pearl of wisdom: You could lose the menace response with ipsilateral facial nerve lesion in the brainstem, but you would NOT have the wandering and head pressing

A 15-year-old cat is presented with a three week progression of wandering in circles to the left, standing in corners, and dull mentation. Conscious proprioceptive deficits (knuckling) can be elicited in the right thoracic and right pelvic limbs. There is a decreased menace response on the right, but palpebral and pupillary light reflexes are normal in both eyes. The rest of the examination is normal. A brain tumor is suspected. Where is the most likely location of the lesion? a. RIght brainstem b. Right cerebrum c. Left brainstem d. Left cerebrum e. Cerebellum

a. Oral methimazole Hyperthyroid patients should be treated with oral methimazole initially to unmask any occult renal insufficiency that could have been hidden by the hyperthyroidism. This is the safest initial treatment in that respect since its effects are reversible. Radioactive iodine therapy and thyroidectomy are irreversible. Sodium levothyroxine is thyroid hormone supplementation used in dogs with hypothyroidism.

A 15-year-old female spayed Siamese cat is diagnosed with hyperthyroidism based on clinical signs of weight loss and an unkept hair coat, in conjunction with a palpable thyroid nodule and elevated serum T4, CBC, chemistry panel, and thoracic radiographs are within normal limits. Which is the most appropriate initial therapy? a. Oral methimazole b. Thyroidectomy c. Radioactive iodine (I-131) therapy d. Sodium levothyroxine

a. Doxycycline

A 2 year-old intact male domestic shorthaired cat is being evaluated because of lethargy. The cat has a fever. Laboratory studies show anemia. Microscopic examination of stained blood films shows dark-staining, round-to-rod-shaped, nonrefractile bodies on the surfaces of numerous red blood cells. Which of the following is the most appropriate therapy? a. Doxycycline b. Griseofulvin c. Penicillin d. Trimethoprim-sulfonamide e. Vincristine

a. Upper GI obstruction Bacause of this cat's history and blood results, an upper GI obstruction is highly suspected. Hypochloremic metabolic alkalosis is a classic finding in a pet with an upper GI obstruction and should be ruled out first, especially in a young cat with these clinical signs. Acute renal failure is unlikely since the azotemia appears to be pre-renal (the urine is hyperconcentrated due to dehydration). Hypoadrenocorticism, or Addison's disease, is very unlikely in a cat. In a pet with Addison's the most common electrolyte finding would be an elevated potassium level A urethral obstruction would likely cause a post-renal azotemia and hyperkalemia. On physical exam, a large and painful bladder would be palpated. A cat having urethral obstruction going on for this period of time would be very critical and have other clinical signs. Renal lymphoma would be a potential cause of acute renal failure wh

A 2-year-old MN DSH presents for anorexia and vomiting of 2 days duration. Physical examination reveals the cat is dehydrated, but otherwise no abnormalities are noted. Bloodwork shows the following: BUN 55mg/dL (19-34mg/dL), creatinine 3.8mg/dL (0.9-2.2mg/dL), sodium 135 mEq/L (146-156mEq/L), potassium 3.1 mEq/L (3.7-6.1 mEq/L), chloride 85 mEq/L (115-130 mEq/L), TCO2 38 (13-21 mEq/L), HCT 60% (30-45%). Urinalysis: USG 1.058 (>1.035), negative sediment You started the cat on 0.9% NaCl IV to treat the dehydration. Based on this history and blood results, what is your top differential diagnosis? a. Upper GI obstruction b. Acute renal failure c. Hypoadrenocorticism d. Urethral blockage e. Renal lymphoma

d. Abdominal radiographs, upper GI obstruction Hypochloremia in a young vomiting anorexic animal is most suggestive of an upper gastrointestinal obstruction. The next best diagnostic tool in this case would be abdominal radiographs followed by a barium series if indicated. While viral diseases and food allergies can also trigger lethargy, dehydration, and vomiting in a young cat, these are less likely. These should be investigated after an upper GI obstruction has been ruled out

A 2-year-old MN domestic short-hair named Chopper presents with a 2 day history of lethargy and not eating. The cat vomited twice this morning. Temperature is normal and bloodwork is all within normal limits except the chloride is 92 mEq/L (115-130 mEq/L) and sodium is 138 mEq/L (146-156mEq/L). Which of the following tests would be the next best step for the most likely diagnosis? a. Feline coronavirus titers, feline infectious peritonitis b. Hypoallergenic diet trial, inflammatory bowel disease c. FeLV/FIV test, feline leukemia virus d. Abdominal radiographs, upper GI obstruction e. Canine parvovirus snap test, feline panleukopenia virus

e. A string foreign body

A 2-year-old cat has a two-day history of anorexia and vomiting. Abdominal radiographs show that the intestine sppear bunched up in accordion-like pleats. Which of the following is the most likely cause? a. Intestinal adenocarcinoma b. Intestinal lymphosarcoma c. Intestinal parasites d. Intussusception e. A string foreign body

a. Feline infectious peritonitis This poor cat is infected with the dreaded wet form of feline infectious peritonitis. If you think about the pathogenesis of the virus, then these clinical signs make a lost of sense. With FIP, the classical lesion is pyogranulomatous vasculitis due to antigen-antibody complexes deposition in the venular endothelium, which results in pleural and peritoneal effusion. The CBC findings are common for FIP but not too specific. In greater than 50% of cats with the wet form of FIP, there will be hyperproteinemia. In greater than 70% of cats with the dry form of FIP, there is hyperproteinemia

A 2-year-old female DSH cat presents for weight loss, anorexia, dyspnea, and lethargy. She was previously treated with antibiotics but is still febrile on physical exam. You detect pleural effusion and notice that the abdomen is distended. On CBC there is a non-regenerative anemia, neutrophilia, and lymphopenia. On chemistry, there is hyperproteinemia and slight elevation in liver enzymes. What is your primary differential? a. Feline infectious peritonitis b. Feline leukemia virus c. Feline immunodeficiency virus d. Feline calicivirus

c. Feline infectious peritonitis This poor cat is infected with the dreaded wet form of feline infectious peritonitis. If you think about the pathogenesis of the virus, then these clinical signs make a lot of sense. With FIP, the classical lesion is pyogranulomatous vasculitis due to antigen-antibody complexes depositing in the venular endothelium, which results in pleural and pertioneal effusion. The CBC findings are common for FIP, but not too specific. In greater than 50% of cats with the wet form of FIP, there will be hyperproteinemia. In greater than 70% of cats with the dry form of FIP, there is hyperproteinemia

A 2-year-old female DSH cat presents for weight loss, anorexia, dyspnea, and lethargy. She was previously treated with antibiotics but is still febrile on physical exam. You detect pleural effusion and notice that the abdomen is distended. On CBC, there is a non-regenerative anemia, neutrophilia, and lymphopenia. On chemistry, there is hyperproteinemia and s slight elevation in liver enzymes. What is your primary differential? a. Feline calicivirus b. Feline leukemia virus c. Feline infectious peritonitis d. Feline immunodeficiency virus

a. Fecal-oral transmission The case describes a cat with the wet form of FIP. The primary source of infection of the FIP virus is due to fecal-oral transmission. Eventually, feline corona virus mutates to a virulent feline infectious peritonitis virus which is able to multiply in macrophages. Ideally, a strong cell-mediated immune response eliminates the virus, but in some cats, the infection can become latent and is reactivated with stress. If a cell-mediated immune response is not mounted, pyogranulomatous vasculitis will occur due to the deposition of antigen-antibody complexes in the venous endothelium. Complement-mediated inflammation results in pleural and peritoneal effusion (wet form) and partial cell mediated immune response results in slow viral replication with granuloma formation (dry form).

A 2-year-old female DSH cat presents for weight loss, anorexia, dyspnea, and lethargy. She was previously treated with antibiotics but is still febrile on physical exam. You detect pleural effusion and notice that the abdomen is distended. On CBC, there is a non-regenerative anemia, neutrophilia, and lymphopenia. On chemistry, there is hyperproteinemia and slight elevation in liver enzymes. Spread of the virus that leads to this disease usually occurs from which route of transmission? a. Fecal-oral transmission b. Respiratory secretions c. Aerosol transmission d. Mucous membranes

d. Exploratory laparotomy with linear foreign body removal Linear foreign bodies are surgical emergencies. The string is often anchored at the base of the tongue. The peristaltic activity of the stomach and intestines around a string causes bunching of the intestines. With peristaltic movement, the string will often saw through the intestines, leading to perforation and peritonitis. Induction of emesis, endoscopy, or release of the string would be unlikely to relieve the bunching of the intestine

A 2-year-old male neutered domestic short hair cat presents for acutely being anorexic, lethargic, and not passing any stool. The owner reports that the cat was playing with dental floss 2 days ago and may have swallowed it. On physical exam, the cat is depressed, febrile with a temperature of 104.3F/40.2C, and resists abdominal palpation. On oral exam, you see the one end of the dental floss is attached to the base of the ventral side of the tongue. Abdominal radiographs show clumped intestines, loss of serosal detail, and an irregular gass pattern in the abdomen. What should you recommend to the owner? a. Release the dental floss and allow it to pass through the intestinal tract on its own b. Induction of emesis with apomorphine c. Endoscopy for linear foreign body removal d. Exploratory laparotomy with linear foreign body removal e. Induction of emesis with xylazine

d. Polycystic kidneys Persian cats are predisposed to developing polycystic kidneys. The cat in this question is not yet clinical for the disease. Renal lymphoma and amyloidosis would not typically show fluid-filled structures in the kidneys on ultrasound. Bilateral hydronephrosis would show dilatation of the renal pelvis, not fluid-filled structures within the renal parenchyma

A 3-year-old female Persian cat presents for routine physical exam. The cat seems to be doing well at home and has no clinical signs. Abdominal palpation reveals bilaterally enlarged kidneys. Ultrasound exam shows multiple, large, fluid-filled structures in the parenchyma of the kidneys. What is the most likely diagnosis? a. Amyloidosis b. Renal lymphoma c. Hydronephrosis d. Polycystic kidneys

a. Feline infectious peritonitis With FIP, the classical lesion is pyogranulomatous vasculitis due to antigen-antibody complexes depositing in the venular endothelium, which results in pleural and peritoneal effusion. Usually, the effusion has a high proein (5-12g/dL) and is low in cellularity. The predominant cells are neutrophils. The fluid may be clear to yellow and contain fibrin clots. Give n the low cellularity, neoplasia is lower on your list. FeLV and FIV do not usually present with pleural effusion

A 3-year-old male DSH cat presents to your clinic for dyspnea. On physical exam, the cat is febrile. You identify pleural effusion and obtain a sample. The fluid has a protein of 9g/dL and is thick. The cellularity is low and the predominant cell type is neutrophils. What is your primary differential? a. Feline infectious peritonitis b. Feline immunodeficiency virus c. Thyoma d. Lymphoma e. Feline leukemia virus

d. MRI of the brain This cat has clinical signs most consistent with acromegaly (weight gain, prognathism, organomegaly, and diabetes mellitus that is difficult to control). This condition results from a growth hormone (GH) secreting pituitary tumor. GH or insuline-like-growth factor (IGF) assays can be diagnostic but are not offered by many veterinary laboratories. Advanced imaging techniques are the most reliable way to diagnose a pituitary mass in this instance. This cat is not likely to be hyperthyroid due to the history of weight gain. Hyperadrenocorticism as a cause of insulin-resistant diabetes mellitus is less likely give n the cat's other clinical sings, neverthless, adrenal function testing such as an ACTH stimulation test or dexamethasone suppression test to rule out hyperadrenocorticism should be considered

A 3-year-old male castrated c=domestic short hair cat presents to you for weight gain, polyuria and polydipsia over the course of 6 months. His previous veterinarian made a diagnosis of diabetes and has tried to control this with insulin but has been successful. On examination, you detect prognathism, hepatomegaly, and a grade III/VI heart murmur. You suspect that the cat may have an underlying disorder that is contributing to his poorly-controlled diabetes. Which of the following diagnostic tests will be most helpful in confirming your suspicion? a. Serum insulin:glucose ratio b. Serum folate and cobalamin levels c. Abdominal ultrasound d. MRI of the brain e. serum T4 levels

b. Eosinophilic granuloma Eosinophilic granulomas or linear granulomas are usually linear in shape and pink-yellow in color. Typically, they don't crust and are not usually pruritic. Notoedres acariasis, insect bite hypersensitivity, and dermatophytosis more characteristically cause pruritis, crusting, and often affect the head, neck and ears.

A 3-year-old male castrated domestic short hair cat presents for crusting and pruritus around the face, neck, and ears. Which of the following should be least likely on your list of differential diagnosis? a. Notoedres acariasis b. Eosinophilic granuloma c. Dermatophytosis d. Arthropod bite hypersensitivity

b. Gently slide a thermometer alongside the prolapsed tissue. If it only goes in a short distance, this is a prolapse. If it goes in a significant distance, this would be an intussusception The best way to distinguish is to pass a blunt probe between the protruding mass and anus. If the probe contacts a fornix within a couple of cm, it is likely a rectal prolapse, but if it passes 5-6cm easily then an intussusception is most likely. A dextrose solution may help to reduce edema associated with the tissue, but would not in itself reduce the protrusion. Exploratory surgery would be indicated and is the recommended treatment for intussusceptions. This would be a definitive way of determining if it is a prolapse vs. intussusception, but this is the more invasive way of determining and is not the only way. If it is a rectal prolapse, more conservative therapy should be considered before surgery.

A 3-year-old male neutered Siamese cat has 6cm of tissue protruding from his anus. The owner states when they woke up he was like that. She states he had no diarrhea, constipation, vomiting, or straining that they are aware of but they have been out of town for the last 2 days. You are unable to reduce the lesion initially with gentle manipulation. The tissue appears grossly healthy and no necrosis is noted. You are not sure if this is a prolapsed rectum or a colorectal intussusception. How can you tell? a. Do a barium enema and see if the barium goes into the colon. If it does, it is a prolapse b. Gently slide a thermometer alongside the prolapsed tissue. If it only goes in a short distance, this is a prolapse. If it goes in a significant distance, this would be an intussusception c. Pour dextrose over the tissue and see if it reduces. If it reduces, it is a prolapse. d. The only way to definitively tell is by exploratory surgery e. Take a left lateral radiograph of the abdomen and rectal region

b. Left side of the neck Esophagostomy tube placement is typically performed under general anesthesia. The patient is placed in right lateral recumbency and the left lateral and ventral cervical area is clipped and an aseptci preparation performed. The feeding tube is pre-measured from the site of placement to roughly the 12-13th rib. A curved hemostat or right -angle forceps is introduced through the mouth and into the proximal esophagus. Later, pressure is applied to press the tip of the forces against the skin caudal to the ramus of the mandible. A stab incision is made through the skin and subcutaneous tissue over the tip of the forceps at the LEFT SIDE of the neck. The tip of the forceps is then pushed through the small stab incision. Some surgeons prefer to push the forceps through the esophagus and subcutaneous tissue to avoid trauma to the vessels and nerves on the side of the neck. With adequate lateral pre

A 3-year-old male neutered domestic shorthair cat who is extremely obese presents with hepatic lipidosis. His ALP is 520 U/L (0-45 U/L) and his ALT is 305 U/L (25-97 U/L). He stopped eating when his owner left him in a kennel when he went out of town. You need to place a feeding tube in a addition to his other treatments since the onwer is unable to force feed him and given him medications. You know that nutrition is the maintay of treatment of hepatic lipidosis. Where is the proper location to place his esophagostomy tube? a. Right side of the neck b. Left side of the neck c. Left paracostal region 2-3cm caudal and parallel to the last rib d. An esophagostomy tube is not appropriate for this cat due to his obesity and risk of anesthetic e. Ventral region of the neck

e. Thrombus at the aortic bifurcation (saddle thrombus) Maine Coon cats are predisposed to development of hypertrophic cardiomyopathy at a young age. Consequently, left atrial enlargement predisposes to atrial thrombus formation, and these clots frequently lodge in the arterial supply to the hindlimbs. The trifurcation is where the aorta divides into the tow external iliac arteries and the common origin of the internal iliac arteries. Classic findings due to a clot at the aortic trifurcation include posterior paresis/paralysis, hindlimb pain, cyanotic nailbeds, absent femoral pulses, and a firm leg musculatue. Other signs of cardia disease/failure (murmur or pulmonary edema) are often but not always evident at presentation Neither cord lesions, tetanus, nor toxicities should cause the vascular compromise evident on this cat's physical exam

A 3-year-old, indoor-only, male castrated Maine Coon cat is presented to you for acute-onset of hindlimb pain and paresis. Physical examination reveals a grade III left parasternal heart murmur and minimal movement in the hind legs. Femoral pulses are bilaterally absent, and the hindlimb toes are cold to the touch. The cat is tachypenic, but lung sounds are normal. What is the most likely diagnosis? a. Tetanus b. Lymphoma of the spinal cord c. Chlopyrifos toxicity d. T3-L3 intervertebral disc disease e. Thrombus at the aortic bifurcation (saddle thrombus)

c. Esophagotomy tube feeding The cat described likely has hepatic lipidosis. Cats that are greater than 2 years of age and obese have the greatest risk for hepatic lipidosis. Often these cats are indoor-only and have had a recent stress in their life. An obese cat that is not eating with the above symptoms is most likely to have hepatic lipidosis. An ALP elevation that is greater in magnitude than GGT is also suggestive of hepatic lipidosis. A bile duct obstruction, cholangiohepatitis, lymphoma, and FIP are other differential but are less likely with the given information

A 4-year-old domestic short haired cat presents for anorexia and weight loss of 1 week. Physical exam reveals a body condition score of 7/9, jaundice of the skin and sclera, and dehydration of 4%. Temperature is normal. Bloodwork shows: -ALT=303 (25-97 U/L) -GGT=1.8 (0-6 U/L) -ALP =1170 (0-45 U/L) -Bilirubin=3.0 (0-0.1 mg/dL) Radiographs shows an enlarged liver. What is the most important treatment for the cat's likely diagnosis? a. Oral clavulanic acid and amoxicillin for 4 weeks b. Parenteral vitamin K injections c. Esophagotomy tube feeding d. Oral S-adenosylmethionine for at least 1 month

b. Acetylcysteine and S-adenosylmethionine Acetaminophen toxicity in cats usually occurs when owners administer the drug, unaware of its significant potential toxicity in cats. In this case, the cat's clinical signs are most consistent with acetaminophen toxicity based on the Heinz body anemia that is present. Cats can die from oxidative damage and methemoglobinemia within 1-2 days of ingestion. It may also be associated with hepatotoxicity in cats, although this is seen more frequently in dogs. Recall that cats are particularly sensitive to acetaminophen because they have decreased glucuronyl transferase activity which conjugates acetaminophen to glucuronic acid for excretion. As a result, 50-60mg (a single tablet) may be fetal for a 4-5kg cat. Treatment should consist of toxin removal if possible by inducing emesis in some cases. As the cat in this case is already vomiting, this may not be necessary. Activated

A 4-year-old male Manx cat presents to you because the owners found an empty, opened pill vial in the bathroom and the cat vomited. On physical exam, you note ptyalism and facial edema. The cat's mucous membranes are pale and slightly icteric. You perform a blood smear and detect Heinz bodies in erythrocytes. The cat's packed cell volume (PCV) is 26% (30-45%). The owners provide you a list of the medications in the medicine cabint which are acetaminophen (Tylenol), finasteride (Propecia), enalapril (Vasotec), and omeprazole (Prilosec). What treatments should you institute for this cat? a. Emesis and methylene blue b. Acetylcysteine and S-adenosylmethionine c. Prednisone and amoxicillin d. Activated charcoal and whole blood transfusion

c. Acetylcysteine and S-adenosylmethionine Acetaminophen toxicity in cats usually occurs when owners administer the drug, unaware of its significant potential toxicity in cats. In this case, the cat's clinical signs are most consistent with acetaminophen toxicity based on the Heinz body anemia that is present. Cats can die from oxidative damage and methemoglobinemia within 1-2 days of ingestion. It may also be associated with hepatotoxicity in cats, although this is seen more frequently in dogs. Recall that cats are particularly sensitive to acetaminophen because they have decreased glucuronyl transferase activity which conjugates acetaminophen to glucuronic acid for excretion. As a result, 50-60mg (a single tablet) may be fatal for a 4-5 kg cats. Treatment should consist of toxin removal if possible by inducing emesis in some cases. As the cat in this case is already vomiting, this may not be necessary. Activated

A 4-year-old male Manx cat presents to you because the owners found an empty, opened pill viral in the bathroom and the cat vomited. On physical exam, you note ptyalism and facial edema. The cat's mucous membranes are pale and slightly incteric. You perform a blood smear and detect Heinz bodies in erythrocytes. The cat's packed cell volume (PCV) is 26% (30-45%). The owners provide you a list of the medications in the medicine cabinet which are acetaminophen (Tylenol), finasteride (Propecia), enalapril (Vasotec), and omeprazole (Prilosec). What treatments should you institute for this cat? a. Prednisone and amoxicillin b. Activated charcoal and whole blood transfusion c. Acetylcysteine and S-adenosylmethionine d. Emesis and methylene blue

a. Cats have naturally-occurring antibodies against the blood group agntigen they lack Cats have naturally-occurring antibodies against the blood group antigen they LACK. Feline blood groups include A, B, and AB (rare). Type A cats have naturally-occurring anti-B antibodies without any previous exposure to type B antigens. Type B cats have naturally-occurring anti-A antibodies. Type AB cats do not have antibodies to either group. Type B cats that receive type A blood are particularly susceptible to fatal transfusion reaction as a result of severe acute intravascular hemolysis. Type A cats that receive TYpe B blood have a weaker reaction that commonly results in extravascular hemolysis. Type AB cats should receive Type AB or Type A blood. Unlike dogs, there are no universal donors in cats.

A 4-year-old male, neutered cat is presented with a history of severe lethargy and inappetance. The cat's mucous membranes are extremely pale and his PCV at presentation is 9% [Normal=30-45%). A blood transfusion is indicated. Which one of the following choices best explains the rationale for blood typing prior to transfusion in cats? a. Cats have naturally-occurring antibodies against the blood group agntigen they lack b. Blood typing is not necessary. Cats are universal receivers and can receive any blood type c. Cats that are negative DEA1.1 are more likely to have a reaction to DEA 1.1 positive blood. d. If donor is TYpe A, it is a universal donor and recipient typing is not needed e. Blood typing will determine if recipient cat is Rh factor negative or Rh positive

b. Aortic thromboembolism

A 5-year-old cat is being evaluated because of a sudden onset of paralysis of the rear legs. Femoral pulses are absent, and the rear legs are cold and painful. Spinal reflexes of the rear legs are absent. Which of the following is the most likely diagnosis? a. Aberrant parasitic migration b. Aortic thromboembolism c. Fibrocarilaginous emboli d. Intervertebral disk herniation e. Trauma to the spinal cord

d. Megaesophagus Radiographic interpretation: The intrathoracic esophagus is diffusely distended, containing fluid, the cardiopulmonary structures appear to be within normal limits There is mild malangulation between the 2nd and 3rd sternebra. The caudal, cervical and cranial thoracic vertebra have marked ventral spondylosis present. Within the viewable abdomen, a large volume of gas is present within the GI tract. There is marked osteophytosis of the elbows bilaterally. Megaesophagus is uncommon in cats. Nearly half of the causes of megaesophagus in cats are idiopathic. It could also be secondary to neuromuscular disease (e.g., myasthenia gravis, dysautonomia). Congenital myasthenia gravis can be seen in Siamese cats

A 6-year old male neutered domestic shorthair cat is presented with 4-month history of weight loss and regurgitating undigested food immediately after eating. Which one of the following abnormalities is apparent on the patient's radiographs? a. Pneumonia b. Pulmonary mass c. Left atrial enlargement d. Megaesophagus e. Diskospondylitis

e. Calicivirus Typically, feline calicivirus is an upper resrpiatory pathogen that may cause oral ulceration +/- conjunctival chemosis. However, outbreaks of highly virulent and often lethal feline calicivirus infections have been see, This is frequently referred to as "Virulent systemic feline calicivirus (VS-FCV)." These infections are characterized by the signs described above; typically cats develop a severe acute upper respiratory tract disease first, followed by characteristic signs of cutaneous edema and ulcerative lesions on the skin and paws. Edema is located mainly on the head and limbs. Some cats may be jaundiced due to hepatic necrosis and/or pancreatitis. Thromboembolism and coagulopathy caused by DIC may be observed including petechiae, ecchymoses, epistaxis or hematochezia

A 5-year-old cat presents to you with an acute presentation of marked upper respiratory signs as well as ulcerative and edematous lesions of the skin on the head and limbs. Which of the following viruses can cause this type of syndrome in cats? a. Feline leukemia virus b. Feline immunodeficiency virus c. Panleukopenia d. Feline viral rhinotracheitis e. Calicivirus

This is case of Blastomycosis, caused by Blastomyces dermatitidis, a systemic fungal disease. It is a dimorphic soil fungus that is found in the Mississippi, Missouri, and Ohio River valleys and also the mid-Atlantic states and the Canadian provices of Quebec, Manitoba and Ontaria. Infection occurs primarily through inhalation. This is a systemic infection that can cause damage to any organ or system including the central nervous system but the most common clinical signs and physical exam findings include: Depressioon, anorexia, weight loss, fever, lameness, lymphadenopathy, harsh lung sounds, draining skin lesions, chorioretinitis, uveitis, and cough. Clinical signs may be suggestive of disease, but definitive diagnosis depends on identification of the organism. As seen in the image. Blastomyces appears as a round to ovoid yeast measuring 8-25 um. They have single broad-based buds and stains typically stains blue

A 6 year old male Siamese cat presents to you for evaluation of respiratory difficulty, chronic cough, lethargy, and decreased appetite. On your exam, the cat is quiet and 5% dehydrated. T-103.4F (39.7C), HR-180 bpm, RR-40. Body conditiooon score is 3/9. The cat has harsh lung sounds and coughs occasionally. You note mild peropheral lymphadenopathy/ A CBC shows: HCT- 39% (30-45%), WBC-17,300/uL (5,500-19,500/uL), Bands-600/uL (0-300/uL), Lymphocytes-1,900 (1,500-7,000/uL), Monocytes-600/uL (0-900/uL), Eosinophils-400/uL (0-800/uL) You perform a needle aspirate from an enlarged lymph node. A representative cytology is shown in the image below. Which of the following is the most appropriate treatment?

b. Fungal culture of hair from the lesions

A 6-month-old kitten has several rapidly expanding circular patches of alopecia. The owner's daughter has developed a lesion on her hand. Wood's light examination shows no fluorescence. Which of the following is the most appropriate next diagnostic test to perform on the kitten? a. Bacterial culture of hair from the lesions b. Fungal culture of hair from the lesions c. Fungal serology d. Histopathologic and cytologic examination of biopsy specimens of the lesions e. Microscopic examination of deep skin scrapings from the lesions

b. Nutritional secondary hyperparathyroidism

A 6-month-old kitten is being evaluated because of abnormal curvature of the legs. Radiographs of the skeleton show multiple healing fractures and generalized osteopenia. Which of the following is the most likely cause? a. Disuse seteopenia b. Nutritional secondary hyperparathyroidism c. RIckets d. Scurvy e. Toxic soteopenia

c. Microscopic examination of aural exudate

A 6-month-old kitten is being evaluated because of intense scratching around the ears and frequent shaking of the head. Examination shows bilateral, circumaural, cutaneous excoriations and thick reddish brown crusts in the external ear canal. Which of the following is the most appropriate diagnostic test? a. Bacterial culture of aural exudate b. Fungal culture of aural exudate c. Microscopic examination of aural exudate d. Microscopic examination of deep scraping of cutaneous lesions e. Wood's light examination

b. Diaphragmatic hernia Diaphragmatic hernia is most likely to cause dyspnea with GI sounds auscultable in the chest

A one-year-old intact male domestic shorthair cat is presented with dyspnea after vehicular trauma. On physical examination, the cat is open-mouth breathing with muffled heart sounds and borborygmi heard on pulmonary auscultation. Which one of the following choices is the most likely diagnosis? a. Cardiac tamponade b. Diaphragmatic hernia c. Posterior lung lobe torsion d. Flail chest e. Pheumothorax

b. Immunosuppressive therapy and a change to a hypoallergenic diet This cat's image and clinical history of a disappearing and recurring lip ulcer is consistent with an indolent ulcer, part of the eosinophilic granuloma complex. These are mostly found on the upper lip of cats. These lesions can also extend into the oral cavity and can be found at the tongue base or hard palate. Typical treatment involves high doses of corticosteroids and identifying possible underlying allergic disease. Sometimes, antibiotics are indicated if severe infection is present. A hypoallergenic diet can be helpful if food allergy is the cause. Many times, the etiology is unknown but allergy is suspected. Squamous cell carcinoma is also differential; however, in a cat that has a lesion that comes and goes, this is much less likely. Viral infections in cats usually present with upper respiratory and/or ocular signs.

A 6-year-old female spayed indoor only domestic medium hair presents with a large raw lesion on its upper left lip. The owner reports the lesion has come and gone over the past few years. Which of these treatments would likely be most helpful? a. Topical trifluridine and systemic clindamycin b. Immunosuppressive therapy and a change to a hypoallergenic diet c. Marginal surgical excision and radiation therapy. d. Wide surgical excision and chemotherapy

b. Echocardiography

A 6-year-old neutered male cat is being evaluated because of lethargy and depression. On cardiac examination, a grade III/VI murmur and a gallop rhythm are detected. Heart size appears normal on a radiograph. Which of the following tests will provide the most accurate cardiac anatomic information? a. Aortic angiography b. Echocardiography c. Electrocardiography d. Pericardiography e. Pulmonary angiography

a. Bethanechol

A 6-year-old neutered male cat was treated for a urethral obstruction three days ago but continues to have a full bladder that can be easily expressed. Which of the following drugs is most likely to help treat this cat's problem? a. Bethanechol b. Ephedrine c. Phenoxybenzamine d. Phenylpropanolamine e. Propantheline

c. Left-sided congestive heart failure The cardiac silhouette is tall suggestive of left ventricular enlargement and there is a diffuse unstructured interstitial pattern in the caudoventral and cranial lung fields. The pulmonary vasculature is also dilated. Remember that cats in heart failure can have edema distributed in patchy infiltrates throughout the lungs, in contrast to dogs where it is more typically in the perihilar region. Asthma is less likely as no clear bronchial marking are seen and because of the infiltrate. Neoplasia is a consideration, but doesn't explain the cardiac enlargement. Pneumonia could also cause this pattern of infiltrate (although it is more typically cranioventral), but also does not explain the cardiomegaly. There is no evidence of tracheal collapse, which is exceedingly rare condition in the cat.

A 7-year-old male castrated cat presents for respiratory distress with open-mouth breathing. You initially place him in oxygen and obtain thoracic radiographs when he is stabilized. The films are shown in photos. What is your diganosis? a. Feline asthma syndrome b. Tracheal collapse c. Left-sided congestive heart failure d. Neoplasia e. Pneumonia

b. Affected teeth should be extracted and it is likely that other teeth will be affected in the future As described in this case, feline odontoclastic resorptive lesions commonly affects cats with increasing incidence as cats age. One of more lesions are found in about 50% of the domestic cat population over 5 years old. Lesions are often seen at the buccal surface of premolars and the first molar teeth at the gingival margin. Canine teeth can also be affected but usually lesions occur in the roots and the crown may appear normal. There have been many theories about the cause of these lesions including a relationship to plaque-induced imflammation, microfractures of the cemental surface, and mineral deficiencies but most of these theories are no longer supported. It is now believed that abnormal formation or mineralization of cementum results in cemental resorption. There may be a relationship to high levels of vit

A 7-year-old male castrated cat presents to you for difficulty eating. On examination, you see that there are dental lesions on the buccal surfaces of several premolar and first molar teeth. The ca t shows signs of discomfort when you palpate around these teeth and the surrounding gingiva appears inflamed. You suspect that the cat has ondontoclastic resorptive lesions. You perform dental radiographs which show evidence of endodontic necrosis. Which of the following is the most appropriate treatment recommendation? a. Administration of an analgesic may provide relief until the lesion spontaneously resolves b. Affected teeth should be extracted and it is likely that other teeth will be affected in the future c. Affected teeth should be extracted and Vitamin D supplementation may reduce the likelihood of development of similar lesions in other teeth d. Affected teeth should be treated by removal of the crown and coronal part of the root with a dental burr followed by suturing the gingiva across the root e. Dental extraction is a less expensive option, but restorative dental techniques are effective at stopping progression of disease in most cats

b. Fluid therapy to correct hydration, medical management with psyllium and cisapride Based on the radiographs, you should have diagnosed this cat with megacolon and colonic impaction. The components of treatment for this condition are to achieve and maintain optimal hydration, remove impacted feces, dietary fiber and/or laxative treatment and use of colonic prokinetic agents. An enema is probably indicated for this cat but was not part of the answer choices. Cisapride is no longer widely available because of cardiac toxicity in a small number of human patients, but veterinarians obtain cisapride from compounding pharmacies. Psyllium is a non-fermentable fiber that increases the transit rate of ingesta, increasing the frequency of defecation. An enterotomy would be indicated for a small bowel obstruction but not for the colonic obstruction. Antibiotics as well as cimetidine and sucralfate would not be wrong for thi

A 7-year-old male castrated domestic short haired cat presents to you with a chronic history of repeated episodes of vomiting, inappetence, weight loss, and dehydration over the past year. On exam, the cat is 8% dehydrated, HR=180, RR=36. The cat is very agitated and resists palpation. You take the abdominal radiograph shown betlow. Which of the following measures should you recommend for this cat? a. Fluid therapy to correct hydration, medical management with cimetidine and sucralfate b. Fluid therapy to correct hydration, medical management with psyllium and cisapride c. Fluid therapy to correct hydration, medical management with methimazole d. Fluid therapy to correct hydration, medical management with ampicillin and trimethoprim-sulfa e. Fluid therapy to correct hydration, abdominal exploratory surgery for enterotomy and gastrointestinal resection and anastomosis

d. Cryptococcosis Cryptococcosis (Cryptococcus neoformans), a yeast-like fungus which has narrow based budding. In a cat, the respiratory, CNS, eyes, and skin are usually the area that you will see affected. Most of the cases will have nasal involvement characterized by facial swelling, distortion, chronic snuffling and discharge, and granulomatous masses seen coming out of the nostril. Squamous cell carcinoma of the nasal planum will usually present in the form of ulcerative lesions and is seen in older cats. COccidiodomycosis rarely causes lesions in cats and they will present as skin lesions if it does. Cats are rarely affected by Blastomyces or Aspergillus

A 7-year-old spayed DSH cat presents to your clinic with a history of chronic respiratory and ocular disease. You notice nasal discharge, facial swelling, and distortion of the nose (sort of like a "roman nose"). Additionally, there is moderate submandibular lymphadenopathy. Radiographs of the head reveal increased opacity and turbiate destruction. What is the diagnosis? a. Aspergillosis b. Blastomycosis c. Squamous cell carcinoma d. Cryptococcosis e. Coccidioidomycosis

d. Admit patient for fluid therapy and supportive care With the information give, we can be sure that this patient has a mild azotemia which is more than likely pre-renal in origin. With acute renal failure, BUN and creatinine values would be much more increased. There is no indication of an infection and treating blindly with an antibiotic is inappropriate. An exploratory laparotomy would be premature at this time but may be indicated if the patient does not respond to supportive care. It is possible this patient has a mild case of pancreatitis or indigestion. It is best to admit the patient for supportive care and consider performing an abdominal ultrasound

A 9-year-old DSH cat initially presented with a history of decreased appetite and intermittent vomiting. On physical exam, the cat is approximately 5% dehydrate. There is no palpable slip noted in the thyroid region. Blood work performed showed a BUN of 44mg/dL (19-34mg/dL) and creatinine of 2.4mg/dL (0.9-2.2 mg/dL). PCV is 55% (30-45%). GLucose was 180mg (60-120mg/dL). Abdominal radiographs showed a questionable pattern in the area of duodenum but it was not obstructive. A barium series was then performed and did not show any obvious filling defects or delayed emptying. How should you treat this patient? a. Recommend an exploratory laparotomy and obtain biopsies. b. Perform an ultrasound of the urinary tract followed by a nephropyelogram c. Being patient on amoxicillin/clavulonate d. Admit patient for fluid therapy and supportive care

d. Draw blood for a fructosamine level A serum fructosamine should be drawn. Fructosamine concentration is a good indicator of glycemic control of the animal over the past 2-3 weeks. The stress hyperglycemia that occurs in stressed and fractious animals do not affect the results of a serum fructosamine test. Sedating the cat would most likely not eliminate the stress hyperglycemia. Diabetic patients that are prone to stress hyperglycemia can be managed with blood glucose curves taken at home by owner, using commercially available lancets and glucometers

A 9-year-old female spayed domestic short hair presents for a glucose curve. The cat has been diagnosed with diabetes mellitus and has been getting 2 units of lente insulin BID. The cat is fractious and stressed at your clinic. What is the ideal next step? a. Give a sedative to the cat and then perform the glucose curve b. Keep the cat for the day and perform a glucose curve c. Send the cat home and give the owner sedatives to administer to the cat next time they bring the cat in d. Draw blood for a fructosamine level

c. Renal failure Renal failure is the differential most likely to be associated with pu/pd accompanied by decreasing appetite and weight loss. Most cats with diabetes mellitus have normal to increased appetites; diabetic ketacidotic animals can be ill, but usually DKA follows a period of pu/pd, weight loss and normal to increased food intake. Hyperthyroid cats can be pu/pd and have weight loss, but appetite is most often increased. Some EPI animals show pu/pd and most have weight loss, but most are ravenously hungry Most animals with HCM are not pu/pd, and the disease is not routinely closely associated with decreased appetite or weight loss.

A 9-year-old spayed female Siamese cat is presented to you for a 6-month history of polydipsia (the owner has documented water consumption of 110mL/kg/day) and a 1 month history of weight loss and progressively decreasing appetite. Which of the following is a likely differential to explain these clinical signs? a. Hyperthyroidism b. Hypertrophic cardiomyopathy c. Renal failure d. Exocrine pancreatic insufficiency e. Diabetes mellitus

d. Corticosteroids Treating with any agent that is an adultcide may potentially result in embolization, release of antigen, and acute death, making this controversial choice. Corticosteroids work well in reducing inflammation associated with infection and will help alleviate clinical signs. Cats are different than dogs in that heartworms cannot survive as long and the cats are sometimes able to eliminate the worm. Surgical removal has been attempted, but is not a common practice and may also result in acute death

A cat presents with dyspnea and coughing. On a routine blood smear, you find a Dirofilaria immitis microfilaria. How would you teat this cat? a. Thiacetarsamide b. Ivermectin c. Melarsomine d. Corticosteroids e. Surgical removal

d. Diabetic remission Up to 20% of cats may have diabetic remission (sometimes called transient diabetes) characterized by resolution of clinical signs weeks to months after beginning insulin treatment. Clinical diabetes mellitus may or may not recur in the future. Cats with type 2 (non-insulin-dependent) diabetes mellitus may respond to a treatment regimen of weight loss, diet, oral hypoglycemic drugs, and correction of concurrent insulin antagonistic disease; that is, they can respond without injectable insulin Persistent fasting hyperglycemia [blood glucose greater than 200mg/dL and persistent glycosuria are confirmatory for diabetes mellitus, so "incorrect diagnosis" is a wrong answer choice in this case

A cat with a previous diagnosis of diabetes mellitus confirmed by persistent fasting hyperglycemia and persistent glycosuria is presented for a routine check-up. The owner relates that she ran out of injectable insulin tow months ago, but the cat seemed to fine without it, so she stopped giving insulin shots. On physical exam, the cat appears healthy and a dipstick test shows a blood glucose level of 125mg/dL (6.9 mmol/L) [normal 61-132mg/dL (3.5-7.3 mmol/L)]. What is the most likely explanation? a. Type I diabetes mellitus b. Concurrent diabetes insipidus c. Incorrect initial diagnosis d. Diabetic remission e. Insulin resistance

d. BUN and creatinine levels Lilh plant toxicosis is extremely serous and can cause rapid and fatal acute renal failure in cats. If ingestion is suspected, decontamination and aggressive fluid therapy, and monitoring of renal values are recommended immediately. In this case, since 2 days have passed, inducing emesis and administering activated charcoal would not be helpful. What you can do is check renal values (BUN and creatinine) and treat for acute renal failure Ruling out an intestinal obstruction in a cat with this history is reasonable but not the best of the choices given

A client calls and says her cat was chewing on her lily plant two days ago and is now acting very sick. You tell her to bring the cat in immediately so that you can perform which of the following? a. Abdominal radiographs to diagnose intestinal obstruction b. ALT, AST, GGT and total bilirubin level c. Induce emesis and administer activated charcoal d. BUN and creatinine levels

c. Both mother and baby are safe The toxoplasmosis organism causes birth defects in a developing fetus if a mother is infected for the first time in her life while pregnant (ie: IgM positive while pregnant). Toxoplasmosis is not generally dangerous to immune-competent people and a positive IgG result suggests a previous infection. There are challenges to toxoplasmosis testing in pregnant women (false positives). If a pregnant woman is IgM positive, confirmatory test must be done.

A female veterinarian is pregnant and her physician tests her serologically for toxoplasmosis. IgM is negative IgG is positive What is the most appropriate interpretation? a. Mother is safe, baby at risk b. Mother at risk, baby is safe c. Both mother and baby are safe d. Need to re-check in 2 weeks for rising titers e. Both mother and baby at risk

b. Common opening of the pancreatic and common bile ducts into duodenum In cats, there is a common opening of the pancreatic and common bile ducts into the duodenum. It is thought that this may predispose them to ascending cholangitis and pancreatitis after vomiting associated with inflammatory bowel disease, resulting in extrahepatic biliary obstruction. Other possible etiologies of EHBO include neoplasia, biliary stricture, duodenal obstruction, diaphragmatic hernia, and parasitic infection. In all cases, there is a lack of bile entry into the intestinal tract, leading to decreased absorption of fat and fat soluble vitamins, such as vitamin K, potentially resulting in coagulopathies. Except in transient cases that are related to acute pancreatitis, surgical intervention to relieve the obstruction is required as well as appropriate supportive care, antimicrobial treatment, and vitamin K administration.

A five-year-old domestic short-haired spayed female cat is presented with the complaints of anorexia, chronic vomiting, and weight loss. Physical examination reveals icterus, dehydration, and fever. Thickened small intestinal loops are appreciated with abdominal palpation, and serum biochemistry shows increased bilirubin and liver enzymes. The cat is diagnosed with extrahepatic biliary obstruction with accompanying cholengitis and inflammatory bowe ldisease. Which one of the following normal anatomic features may predispose cats to this triad of disease? a. Narrowing of the duodenum distal to the entry of the common bile and pancreatic ducts b. Common opening of the pancreatic and common bile ducts into duodenum c. Enlarged duodenal papilla in comparison to other species d. Decreased proportion of alpha-smooth muscle actin in muscle fibers of the wall of the gallbladder in cats e. Decreased diameter and increased branching of the hepatic ducts in comparison to other species

c. Single serum fructosamine level A fructosamine level measures the number of blood glucose molecules linked to protein molecules in the blood. This value is considered to provide an average of the blood glucose concentration over the past 2 to 3 weeks. Urine dipsticks are not a reliable means of monitoring diabetes. Stress can greatly affect the glucose curves. In some instances the curve is still the best means of monitoring, especially for unregulated patients. The nadir or low point of the glucose curve can occur anytime during the curve, and with PZI is typically 5-7 hours after insulin administration. However, one blood sugar level does not provide a good means of evaluating regulation

A fractious diabetic cat is very stressed when it presents for a blood glucose curve, and the owner does not want to leave the cat in the hospital for the day. The cat is receiving 4 units of PZI insulin SID at home, and was fed and received insulin this morning before coming in to see you. For this cat, what would be a viable alternative to a glucose curve if the cat has been well regulated in the past? a. Serial urine dipsticks by the owner at home to quantify glucosuria b. Check blood glucose once mid-way between insulin injections; this would be equivalent to the nadir c. Single serum fructosamine level d. Stress has no effect on the glucose curve, and the curve is still the best option

b. Full body lyme sulfur dip, itraconazole This cat has a ringworm infection caused by Microsporum canis. This fungi fluoresces blue under a Wood's lamp in 50% of cases. The best treatment for ringworm infection would include a combination topical and oral therapy. Lyme sulfur dip or an antifungal shampoo containing miconazole would be acceptable. Oral antifungals, such as itraconazole or fluconazole are most effective with the least side effects. Providone-iodine scrub has not been shown to be effective against ringworm. Lufenuron is classified as an insect development inhibitor because of its ability to inhibit chitin synthesis, thus in the pas has been said to have some effect against fungal infections. This has been debated and not widely supported as a treatment for ringworm. Doxycycline is an antibiotic and would not be effective in treatment of fungal disease. Anthlet's foot cream (clotrimazole) may have

A middle aged MN stray cat is left on the doorstep of your clinic. The cat has a large dry crusted area of alopecia over his nose. A skin scraping of the area is negative. THe lesion fluoresces under Wood's lamp examination (see image). Which of the following would be the best treatment? a. Doxycycline b. Full body lyme sulfur dip, itraconazole c. Athlete's foot cream (clotrimazole) d. Providone-iodine scrub e. Lufenuron

d. Left sympathetic innervation to the eye This is Horner's syndrome (miosis, ptosis, and enophthalmos), caused by defective ipsilateral sympathetic innervation to the affected eye. Loss of parasympathetic innervation (via the oculomotor nerve) to the eye would cause a dilated pupil. Trigeminal damage would result in decreased sensation to the eye

A seven-year-old cat is presented with miosis, ptosis, enophthalmos, and protruded nictitating membrane involving the left eye. These signs are caused by DAMAGE to which one of the following innervation pathways? a. Right trigeminal innervation to the eye b. Right parasympathetic innervation to the eye c. Left oculomotor innervation to the eye d. Left sympathetic innervation to the eye e. Right vagal innervation to the eye

d. Carpal arthrodesis It is instinctive for cats to always try to land on their feet and as a result, they are predisposed to sustaining hyperextension injuries to the carpal joints when landing from a distance. Disruption of the carpal ligaments carries a guarded prognosis with conservative therapy and stabilization via pancarpal arthrodesis is strongly recommended

A stray cat has been dropped by a good samaritan after having witnessed the cat jump from a 3-story abandoned warehouse. On physical examination the cat has an avulsed lower lip and is very lame on the fron left forelimb. Thankfully the cat had a microchip and the onwers are eventually located. However, they refuse to spend any money on additional diagnostics. What is the best treatment option for the injuries likely sustained to the left forelim. a. Dynamic compression plating of the distal radial fracture b. Cage rest for the scapular luxation sustained c. Reduction of traumatic elbow luxation d. Carpal arthrodesis

a. Pancreatitis Pancreatitis is most likely based on the high trypsin-like immunoreactivity result, plus signs (hypothermia, anorexia) plus bloodwork. Clinical signs of feline pancreatitis are generally vague, and most often include lethargy and anorexia. Icterus occurs in some cats, usually when cholangiohepatitis is also present. Vomiting occurs only in 33% of feline pancreatitis case-patients, and abdominal pain in only ~25%. In CONTRASTS, 90% of DOGS with pancreatitis present with vomiting and anorexia, and 50% have abdominal pain (in severe cases). Abdominal ultrasound typically reveals enlargement and hypoechogenicity of the pancreas. The trypsin-like immunoassay (TLI) is highly specific, but not particularly sensitive in diagnosing pancreatitis. Thus, a negative result doesn't rule out the disease. The feline pancreatic lipase immunoreactivity assay (Spec fPL) is highly specific and has a better sensi

A ten-year-old neutered male cat is presented with vague signs of anorexia, occasional vomiting and lethargy. On physical examination, the patient is somewhat dehydrated. Which one of the following diseases is most likely, based on these findings? a. Pancreatitis b. Feline infectious peritonitis c. Hepatic lipidosis d. Cholelithiasis e. Toxoplasma gondii

c. Diabetes mellitus, Cardiomegaly, Renal disease Think first of Diabetes mellistus in OLDER MALE cats (ave. 10 yr, 90% male) with Feline acromegaly. First presenting sign may be PU/PD, polyphagia of diabetes. WEIGHT GAIN in an unregulated diabetic cats STRONGLY SUGGEESTIVE acromegaly. Think also of Cardiomyopathy (50%; see weakness, dyspnea, systolic murmur, cardiomegaly/CHF, pulmonary edema) Think also RENAL disease (50%; see proteinuria, USpG, 1.015-1.025 (and glucosuria from DM) May see prognathism (long mandible), lameness (esp, cats) marked vertebral spondylosis

Acromegaly is associated with what three diseases in cats? a. Hypoadrenocorticism, Congestive heart failure, Hepatic lipidosis b. Hyperadrenocorticism, Pleural effusion, Lameness c. Diabetes mellitus, Cardiomegaly, Renal disease d. Prognathism, Pulmonary edema, Pulmonary Hypertension e. Vertebral spondylosis, 3rd degree heart block, Glomerulonephritis

e. Ovariohysterectomy

After the initial estrous cycle, a 6-month-old cat has firm enlargement of all mammary glands. Mammary gland hyperplasia is suspected. Which of the following is the most appropriate management? a. Bilateral radical mastectomy b. Estradiol cypionate therapy c. Furosemide therapy d. Megestrol acetate therapy e. Ovariohysterectomy

c. The lesions can be internal and external, so dental radiographs should be taken to further investigate the extent of lesions FORL lesions are usually very painful and are more common now than before. Up to 67% of cats presenting for dental care may be affected. The exact etiology of FORL is not known, but studies have shown an association of FORL and diets low in calcium, magnesium, phosphorus, and potassium. Periodontal disease is also often found in association with FORL. Treatment should include addressing associated periodontal disease and possibly extraction of teeth affected by deep lesions.

An 8-year-old female spayed domestic short hair cat presents for her yearly physical exam. Your oral exam reveals multifocal lesions of absent dental substance on multiple teeth consistent with feline odontoclastic resorption lesions (FORL). Which of the following statements about FORL is true? a. The lesions are usually incidental findings that are not clinically significant and do not need to be treated b. FORL is now a rare clinical finding since commercial diets have become more strictly regulated c. The lesions can be internal and external, so dental radiographs should be taken to further investigate the extent of lesions d. The lesions are usually non-painful

b. Enema, laxatives, Cisapride; fair prognosis Megacolon is well documented in cats. The most common cause in Manx cats is a deformity in the caudal spinal cord. In other cat breeds the most common cause is idiopathic. Other potential causes are pelvic fracture malunion (leading to decreased pelvic canal size), dehydration, GI foreign bodies, and thyroid disease. In mild to moderate cases or first cases of constipation, aggressive re-hydration, increased water consumption, and laxatives can appropriately manage the disease. Cispride is a prokinetic drug which may help with intestinal motility and thus help prevent the formation of fecoliths in cats. Surgical correction with subtotal or total colectomy provides a good to great prognosis in cases which are non-responsive to medical management, although diarrhea and short bowel syndrome can occur weeks to months afterwards. Many cats that are managed medically eventua

An 8-year-old female spayed domestic short hair presented for vocalizing in the litterbox and inappropriate elimination in the house for the past 3 days. The cat is strictly indoors. The cat has previously been healthy and otherwise doing well. On physical exam, the patient is bright, alert, and responsive. Heart rate 180 bpm, RR 40 bpm, T 101.3F (38.5). The cat becomes agitated on abdominal palpation, and a large amount of rock-hard feces is easily palpated. CBC/Chem/T4 are unremarkable. Abdominal radiographs are available for review (see image). What is the best treatment course and prognosis? a. IV fluids and laxatives, great prognosis b. Enema, laxatives, Cisapride; fair prognosis c. Immediate surgical removal of the affected organ; grave prognosis d. No treatment is neede; this is a normal age related change in the cat e. Aggressive fluid therapy, multiple enema; fair to poor prognosis f. surgical evacuation; poor prognosis

c. Hypertrophic cardiomyopathy This is the classic "valentine heart" of feline hypertrophic cardiomyopathy (HCM). Note the high RR and HR. HCM is typically seen in cats 5-7 years old, more often in males. HCM is not always symptomatic; it can be an incidental finding. Look for thickened L. ventricle wall on echocardiography. May see secondary pulmonary hypertension, edema, pleural effusion. DDx includes hyperthyroidism, systemic hypertension, acromegaly, congenital aortic stenosis. Systolic murmur loudest on L between 5th-6th suggests mitral valve insufficiency (left AV). Aortic insufficiency is DIASTOLIC. Remember: Valvular (e.g. mitral, tricuspid) insufficiency is NOT, itself, a disease. It is a description of a leaky or regurgitant valve that results from underlying heart disease. Tricuspid valve murmur (right AV) are loudest on the RIGHT. Taurine deficiency is associated with dilated cardiomyopathy (DCM) in

An 8-year-old male neutered domestic shorthair cat is presented with a 1-month history of progressive exercise, panting, anorexia, weight loss and vomiting. On physical exam, the cat is dyspneic and has weak femoral pulses. On chest auscultation, there is a systolic murmur loudest on the left side between the 5th and 6th intercostal space. A DV chest radiograph looks like the image below. What is the diagnosis? a. Endocardial fibroelastosis b. Aortic insufficiency c. Hypertrophic cardiomyopathy d. Tricuspid insufficiency e. Taurine deficiency

d. Feline calicivirus This is a very typical question. It is important to know some of the major differences in feline upper respiratory infections. We hope you did not fall for the foreign body. If you don't worry there is hope. Feline calicivirus is known for its ability to cause oral ulceration. Clinical signs associated with this virus include lethargy, anorexia, fever, conjunctivitis, sneezing, nasal discharge, oral ulceration, and, rarely nasal ulceration. Feline herpes virus presents with conjunctivitis, coughing, sneezing, fever, anorexia, and rarely oral ulceration. You may also see dendritic ulcers in the eye, which is considered pathognomonic fo the virus. Cats infected with Chlamydophila will show such signs as conjunctivitis and occasional sneezing. It is usually pretty difficult to distinguish these unless you observe the "red flag" clinical sign such as oral ulcers for FCV

An 8-year-old male tomcat presents for having a one week history of progressive lethargy, anorexia, occasional sneezing, conjunctivitis, and nasal discharge. On physical exam, you see several ulcerations in the mouth. What is the most likely diagnosis? a. Feline herpesvirus b. Chlamydophila felis c. Foreign body d. Feline calicivirus e. Bordetella bronchiseptica

d. Strong chance of lifelong FeLV infection 97% of cats positive for feline leukemia virus (FeLV) by IFA remain persistently infected and viremic for life. The FeLV ELISA is more seNsitive than the IFA (fewer false Negs, so trust a NEG test more), so it is a better general screening test with which to start. The FeLV IFA is more sPecific than the ELISA (fewer false Pos, so trust a POS test more) so it is a better confirmatory test for any cats with a positive FeLV ELISA. THREE things to remember about FeLV and vaccinations: 1. FeLC ELISA and IFA tests measure ANTIGEN, not antibody, so FeLV vaccination does NOT interfere with testing 2. Vaccinate FeLV-positive cats yearly against respiratory, enteric viruses with inactivated vaccines. 3. FeLV vaccination for FeLV-negative cats has been associated with sarcoma. Vaccinate LOW on the LEFT hind.

An adopted adult male cat from a shelter with an unknown vaccination history is presented. The cat tests positive for feline leukemia virus (FeLV) infection by IFA. What should the owner be told? a. Possible transient FeLV infection b. Need an ELISA test in 12 weeks to confirm diagnosis c. Cat may be vaccinated for FeLV d. Strong chance of lifelong FeLV infection

e. Strong change of lifelong FeLV infection 97% of cats positive for feline leukomia virus (FeLV) by IFA remain persistently infected and viremic for life. The FeLV ELISA is more seNsitive than IFA (fewer false Negs, so trust a Neg test more), so it is better general screening test with which to start. The FeLV IFA is more sPecific than the ELISA, (fewer false Pos, so trust a Pos test more) so it is a better confirmatory test for any cats with a positive FeLV ELISA. THREE things to remember about FeLV and vaccinations: 1. FeLV ELISA and IFA tests measure ANTIGEN, not antibody, so FeLV vaccination does NOT interfere with testing 2. Vaccinate FeLV-positive cats yearly against respiratory, enteric viruses with inactivated vaccines. 3. FeLV vaccination for FeLV-negative cats has been associated with sarcoma. Vaccinate LOW on the LEFT hind

An adopted adult male cat from a shelter with an unkown vaccination history is presented. The cat tests positive for feline leukomia virus (FeLV) infection by IFA. What should the owner be told? a. Possible transient FeLV infection b. Need an ELISA test in 12 weeks to confirm diagnosis c. Cat may be vaccinated for FeLV d. Need a Western blot test to confirm diagnosis e. Strong change of lifelong FeLV infection

d. 128/191 Sensitivity = 128/191 (67%) a/(a+c) This is a classic example of a 2x2 table in epidemiology, used to compare a new test (the immunochromatographic test kit) to a gold standard test (the CPV test). Draw a 2x2 table, and label the boces a, b, c, d. Sensitivity = 1/(a+c). Feline panleukopenia virus (FPV) is closely related to type 2 canine parvoviruses (CPV-2, CPV-2a, CPV-2b). CPV-2a and CPV-2v have been shown to cause a panleukopenia-like illness in domestic cats.

An immunochromatographic test kit for detection of fecal canine parboviruses (CPV) antigen is being tested in a local cat shelter where as many as 10% of the cats there may have panleukopenia secondary to infection with the canine parvovirus. Here are simulated test results, compared to a gold standard test for CPV. What is the sensitivity of this test kit? a. 734/862 b. 128/734 c. 1575/2309 d. 128/191 e. 1575/1638

a. Dilated cardiomyopath Taurine is an essential amino acid for cats because they cannot synthesize it. Taurine deficiency causes dilated cardiomyopathy (DCM). Since DCN is relatively uncommon otherwise in cats, taurine deficiency should be suspected in any cat with DCM

Cat with taurine deficiency develop which of the following? a. Dilated cardiomyopathy b. Restrictive cardiomyopathy c. Myocarditis d. Hypertrophic cardiomyopathy

c. Biopsy the lymph node and intestine and discuss a poor prognosis with the owner This cat most likely has Feline Infectious Peritonitis, or FIP, which is caused by a mutation of a feline corona virus. Fever, weight loss, and gastrointestinal symptoms are the most frequent presentation. However, this virus may attack multiple organs and can be difficult to diagnose; the only definitive way to dagnose this disease is via histopathology. Clinical symptoms, blood results, and corona virus titers can all be used in combination to help aid in the suspected diagnosis of FIP. Unfortunately, there is no cure for this disease and it is currently considered a fatal disease. DUe to this cat's delining and critical health and strong evidence to support the diagnosis of FIP, euthanasia may be the most humane option. If the owner cannot be reached, however, it is the best option to go ahead and take biopsies while you are in su

Dolly, a 3-year-old female spayed Siamese mix, presents with a history of weight loss over the last months. She is now vomiting occasionally and has a decreased appetite over the last 2 weeks. She has not eaten in the last 2 days. She is current on vaccines. Temperature is 103.7F (39.8C). You can palpate an abdominal mass effect in the mid to caudal abdomen. You believe the cat has a foreign body and are concerned about a possible intestinal perforation. Abdominal tap is negative. Pre-anesthetic bloodwork shows neutrophils 25,000/uL (2,500-12,500/uL), bands 3,000/uL (0-300/uL), globulins 6.9g/dL (2.6-5.1 g/dL). Your x-ray machine is not working today and you recommended an abdominal exploratory. Upon exploratory, the intestines are severely hyperemic and the mesenteric lymph nodes are greatly enlarged. You cannot find a foreign body and no perforations are seen. THere is a small amount of yellow tinged sticky ascites. What do you do? a. Perform a fecal flotation and start sulfadimethoxine for a severe coccidial infection b. Refer to an oncologist for work up of gastrointestinal lymphoma c. Biopsy the lymph node and intestine and discuss a poor prognosis with the owner d. Euthanize the cat on the table since you are unable to reach the owner over the phone about the poor prognosis e. Start the cat on predisolone and hypoallergenic diet for severe inflammatory bowel disease

b. Initial treatment response Feline leukoemia virus seropositive and seronegative cats share similar response rates to initial treatment with cehmotherapy for lymphoma. However, survival times are shorter for cats with FeLV compared with FeLV negative cats. Most US cats with mediastinal, multicentric, or spinal forms of lymphoma are FeLV-positive

Feline leukemia (FeLV) status has been shown to affect many aspects of disease progression and treatment of lymphoma. FeLV seropositive cats with lymphoma are similar to FeLV seronegative cats with lymphoma in which one of the following aspects? a. Freqnency pf spinal lymphoma b. Initial treatment response c. Survival time d. Prevalence of gastrointestinal lymphoma e. Prevalence of mediastinal lymphoma

a. A mixed pleocytosis and elevated protein of CSF analysis with no bacteria Demonstration of the organism, visible as small yeasts surrounded by a large capsule is occasionally seen with Cryptococcus. Serology is also very useful for diagnosis. The latex agglutination test against the cryptococcal antigen is very sensitive and specific and can be used to document disease or monitor therapy. It can be run on serum, CSF, or urine. A mixed pleocytosis is common in many conditions causing CNS signs and is not specific for any of them.

In a cat with CNS signs (ataxia, cranial nerve deficits), which is not a convincing indicator that Cryptococcus is the cause of its signs? a. A mixed pleocytosis and elevated protein of CSF analysis with no bacteria b. A latex agglutination titer of 1:10,000 on serum c. A latex agglutination titer of 1:100 on CSF d. Demonstration of numerous small yeasts with large capsules in a CSF tap

c. Immunofluoresence staining for coronavirus in macrophages in effusion fluid Of the tests listed, immunofluorescence staining for coronavirus in macrophages in effusion fluid has the highest positive predictive value for confirming FIP (reportedly as high as 100%). The NPC of this test is relatively low (57%) so a negative test doesn't rule out FIP. Rivalta's test has a low PPV (58-86% depending on the population tested) and a high NPV (93-97%). The presence of anti-coronavirus antibodies in serum has a PPV=0.44 and NPV=0.90 although high concentrations (>1:1600) has a higher PPV=0.94. The presence of immune-complexes in serum in also not specific for coronavirus with a reported PPV=0.67 and NPV=0.84

Julio is a 1.5-year-old male neutered domestic short hair previously feral but now an indoor only cat. He presents to you today for difficulty breathing and x-rays showed pleural fluid. You remove 200mL of serosanguinous fluid and you are concerned about the possibility of feline infectious peritonitis (FIP). Which of the following tests would be most convincing in confirming a diagnosis of FIP (which has the highest positive predictive value)? a. Presence of anti-coronavirus antibodies in serum b. Rivalta's test c. Immunofluoresence staining for coronavirus in macrophages in effusion fluid d. Presence of immune-complexes in serum measured by ELISA

d. Cisapride There are five main treatment objectives for cats with megacolon. THese include adequate hydration status, removal of impacted feces, laxative therapy, promotility agents for the colon, and dietary fiber. This cat may benefit from Cisapride, which is a benzamide prokinetic drug. Due to the chronic stretching of the colon from the feces impactions, the colon can no longer move the feces out of the body in a normal way. This medication has anecdotally been shown to help cats evacuate feces more efficiently, especially in mild or moderate cases. Some cats may also benefit from a fiber source such as psyllium or canned pumpkin. If the constipation problems continue despite the above mentioned treatments, colectomy should be considered. Prednisolone is used sometimes in treating inflammatory bowel disease, but for megacolon is not indicated. Metronidazole and Sulfasalzine are antibiotics sometimes used fo

Kissy, a 10-year-old female spayed Himalayan, has had chronic constipation problems over the last year and has been diagnosed with megacolon. Today, she presents for vomiting and straining to defecate. She is currently taking Lactulose to help with her bowel movements. You palpate hard feces in the colon. You give her an enema and manually evacuate most of the hard stools. Which of the following medication could also be used to help treat and try to prevent this problem in the future? a. Prednisolone b. Omeprazole c. Sulfasalazine d. Cisapride e. Metronidazole

b. Capillaria aerophila eggs Capillaria ova look similar to Truchuris eggs, but are smaller and have asymmetric terminal plugs. Most cases of Capillaria are asymptomatic, but chronic cough may be seen. Paragonimus is found by fecal sedimentation or rarely by fecal flotation and have a single operculum. Aelurostrongylus larvae are recovered by Baermann

On a cat's annual exam, the owner complains about a chronic cough. You perform a routine fecal flotation and see double operculated eggs with asymmetric terminal plugs. What are they? a. Trichuris ova b. Capillaria aerophila eggs c. Paragonimus ova d. Aelurostrongylus ova

d. Single-layer appositional

Removal of a midjejunum foreign body from a cat requires partial intestinal resection. Which of the following anastomotic techniques is most likely to provide both secure closure and the largest luminal diameter? a. Double-layer appositional b. Double-layer everting c. Double-layer inverting d. Single-layer appositional e. Single-layer inverting

a. Anterior uveitis Rubiosis iridis along with other signs such as aqueous flare, hyphema, hypopyon, keratic precipitates, and decreased intraocular pressure are all suggestive of anterior uveitis which can be caused by a number of infectious, immune-mediated, traumatic, and idiopathic causes

Rubiosis iridis as seen in this cat is a sign of what process in the eye? a. Anterior uveitis b. Chorioretinitis c. Glaucoma d. Lens luxation e. Iris atrophy

e. Anterior uveitis Rubiosis iridis along with other signs such as aqueous flare, hyphema, hypopyon, keratic precipitates, and decreased intraocular pressure are all suggestive of anterior uveitis which can be caused by a number of infectious, immune-mediated, traumatic, and idiopathic causes

Rubiosis iridis as seen in this cat is a sign of what process in the eye? a. Glaucoma b. Lens luxation c. Iris atrophy d. Chorioretinitis e. Anterior uveitis

a. Bluetongue Swollen sore muzzles with mucous membrane erosions, high fevers and lamness suggests infection with bluetongue virus. Bluetongue is almost exclusively seen in sheep, though white-tailed deer, pronghorn antelope and desert bighorn sheep in North America can be severely affected. Rare in cows. REPORTABLE!! Bluetongue is indistinguishable from foot and mouth disease (FMD), so that is a good second choice. But FMD is unlikely in the scenario above because FMD mainly affects pigs and cattle. Ovine progressive pneumonia (OPP) is a chronic disease of wasting and dyspnea in sheep and goats, caused by a lentivirus (retrovirus family), OPP is most common in animals older than 4 years and rarely occurs in sheep and goats under 2-years of age

Several 12-month-old sheep and goats are sick at a petting zoo that has cows, horses, pigs, bison, and white-tailed deer. One of the deer is also affected. The rest of the animals appear well. Affected sheep are febrile, listless, and off-feed, with serous or mucopurulent nasal discharge and rectal temperatures ranging from 105-107.5F (40.5-42C). Physical exams shows swollen muzzles with edema and congestion of the lips, nose, and face with small hemorrhages and ulcers on the mucous membranes. The ulcers appear where the teeth contact the swollen lips and tongue. Two affected animals are lame. What is the diagnosis? a. Bluetongue b. Peste des petits ruminants (PPR) c. Caprine arthritis encephalitis (CAE) d. Contagious ecthyma e. Ovine progressive pneumonia (OPP)

c. Tritrichomonas foetus, Ronidazole Tritrichomonas foetus is a flagellated parasite most commonly found in kittens that have had an unresponsive diarrhea. The parasite can be very difficult to diagnose. It is most often responsive to Ronidazole. Paragonimus is a lung fluke. The eggs are typically passed in feces. Fenbendazole and Praziquantel have been effective against this parasite. Giardia is unlikely if the ELISA is negative as it is a very sensitive test. Taenia is a tapeworm and is not flagellated parasite. It is treated with Praziquantel.

Suzie-Q, a 6-month-old female spayed domestic short hair was recently adopted from the humane society. She has had watery diarrhea since adoption. Her fecal float and Giardia ELISA tests were negative. She was treated with metronidazole with no clinical improvement. You soak a cotton tip swab with saline and swab the rectum You see elongated motile oval shaped protozoan organisms that do not look like GIardia lamblia. What organism might this be and what is the appropriate therapy? a. Cryptosprodium, Clindamycin b. Giardia intestinalis, Fenbendazole c. Tritrichomonas foetus, Ronidazole d. Paragonimus kellicotti, Paraziquantel e. Enterobius vermicularis, Fenbendazole f. Taenia taeniaformis, Praziquantel

c. Rivalta's test Of the tests listed, Ribalta's test has the highest negative predictive value in the diagnosis of FIP. This means that a negative test is likely associated with the cat truly not having the disease. In two separate studies, Rivalta's test has been shown to have a negative predictive value )NPV) greater than 90%. False positives are more commonly seen in older cats and cats with lymphoma or bacterial infections but negative results are relatively convincing compared to most other diagnostic tests for this disease. Rivalta's test involves filling a reagent tube with distilled water and 1 drop of acetic acid (98%). On the surface of this solution, 1 drop of the effusion fluid is added and if the drop disappears and the solution remains clear, the Rivalt's test is negative. If the drop retains its shape, stays attached to the surface, or slowly floats down to the bottom of the tube as a drop, Rivalta'

Sydney is a 1.5-year-old male neutered DSH, previously feral but now an indoor only cat. He has a history of controlled diabetes mellitus and recent bloodwork was within normal limits aside from an elevated an elevated blood glucose of 200mg/dL (60-120mg/dL). He presents to you today for difficulty breathing and x-rays showed pleural fluid. You remove 250mL of serosanguinous fluid and you are concerned about the possibility of feline infectious peritonitis (FIP); which of the following tests would be most helpful in ruling out FIP (which has the highest negative predictive value)? a. Coronavirus antibody detection in effusion fluid b. Reverse-transcriptase polymerase chain reaction (RT-PCR) for coronaviral RNA in serum c. Rivalta's test d. Immunofluorescence staining for coronavirus in macrophages in effusion fluid

c. Low potassium The cat in the image is exhibiting cervical ventroflexion, which is a general sign of weakness. THe most common cause for this presentation is hypokalemia (low potassium), which can be caused by a variety of reasons (such as chronic renal failure). This weakness can also be caused by a number of different problems other than low potassium, including myasthenia gravis; polymyopathies caused by toxoplasmosis, immune-mediated disease, or hyperadrenocorticism; and neuropathies caused by organophosphate poisoning, thiamine deficiency, or botulism.

The cat in the image below for lethargy, depression, and weakness. The cat can't seem to lift his head (as seen in this image). You recommend running a chemistry panel on the cat. What potential finding explains can explain the findings? a. High urea nitrogen b. Low calcium c. Low potassium d. Low phosphorus e. High glucose

c. Chlamydophilia felis The key is to note the severe chemosis which is characteristic of chlamydophilia infections. This upper respiratory tract pathogen does not cause oral ulcers in cats. Herpes virus will cause ocular, dendrictic ulcers, and less commonly cause oral ulcers, and is commonly associated with systemic disease. Calcivirus also causes more systemic disease and can cause oral ulcers. Mycoplasma is not associated with such severe chemosis.

This 8-week-old domestic short hair cat presented with an acute onset of severe chemosis as seen in the photo. The cat is sytemically healthy otherwise with no oral or corneal lesions. Which of the following diseases most commonly causes severe chemosis in the absence of other systemic signs? a. Mycoplasma b. Calicivurus c. Chlamydophilia felis d. Herpesvirus

d. Diarrhea Animals with vascular ring anomalies usually develop clinical signs shortly after being weaned onto solid foods. Vascular ring anomalies will result in constriction of the esophagus which, in turn, does not allow ingesta to travel through the esophagus. These animals will subsequently regurgitate. With enough regurgitation, they can eventually end up with aspiration pneumonia. These animals usually have a varacious appetite; however, they are in poor body condition since they cannot get food past their vascular ring anomaly

Vascular ring anomalies occur rarely in dogs and cats. They are typically diagnosed early in life. Which of the following is not a typical finding in animals with vascular ring anomalies? a. Regurgitation b. Poor Body Condition c. Aspiration Pneumonia d. Diarrhea

b. Hypokalemia

WHich of the following is the most likely cause of generalized polymyopathy secondary to chronic renal failure in cats? a. Hypochloridemia b. Hypokalemia c. Hypomagnesemia d. Hyponatremia e. Hypophosphatemia

a. Expiratory push Asthma causes expiratory dyspnea, or a marked abdominal push seen on expiration with normal inspiration. This occurs as a result of collapse of the lower airways during expiration. This occurs in asthmatics because negative intrathoracic pressure (exerted during expiration) can more easily cause collapse of the thickened and weak bronchial walls. This traps air inside the alveoli. During the next inspiratory cycle, there is decreased fresh air exchange and increasing hypoxemia. Obstructive breathing is characterized by long slow inspirations and can be accompanied by stridor or stertor. This occurs as a result of upper airway disease such as laryngeal paralysis, brachycephalic airway syndrome, etc. Restrictive breathing occurs as a result of pleural space disease and causes short, shallow and rapid breathing Labored breathing is a catch-all phase to describe short rapid and deep breathing. This

What breathing pattern is the hallmark of feline asthema? a. Expiratory push b. Obstructive breathing c. Labored breathing d. Restrictive breathing

e. It is an intact male This is the penis of an intact male cat which demonstrates unique because of the backward-projecting cornified spines on the glans due to the presence of circulating androgens. Penile spins are ABSENT in neutered male cats. The presence of spines can help to diagnose cryptorchid cats. Spines should disappear within 6 weeks of castration. Penile spines help toms fertilize queens by breaking through copulatory plugs. This is helpful in a species in which sperm competition inside the female can be fierce.

What does this finding mean in a cat? a. Nasopharyngeal polyp, cat will be fine after surgical removal b. Lingual polyp, can remove, but recurrence is common c. Cuterebra infestation d. Female in estrus e. It is an intact male

d. 6-7 days Estrus in cat generally lasts 6-7 days (range 1-10 days) Cats are seasonally polyestrous and induced ovulators. The length of feline estrus is affected by whether a male is present. If a male is present, estrus typically lasts 1-4 days. Without a male, estrus lasts 7-10 days and recurs in 2-3 weeks. UNLIKE dogs, the feline estrus cycle is controlled by day length. In North America, cats go through an anestrus period in December and January when day length is less than 12 hours.

What is the average length of estrus in the cat? a. 2-3 days b. 4-5 days c. 9-10 days d. 6-7 days e. 21 days

d. 6-7 days Estrus in the cat generally lasts 6-7 days (range 1-10 days) Cats are seasonally polyestrous and induced ovulators. The length of feline estrus is affected by whether a male is present. If a male is present, estrus typically lasts 1-4 days. Without a male, estrus lasts 7-10 days and recurs in 2-3 weeks. UNlike dogs, the feline estrus cycle is controlled by day length. In North America, cats go through an anestrus period in December and January when day length is less than 12 hours.

What is the average length of estrus in the cat? a. 2-3 days b. 4-5 days c. 9-10 days d. 6-7 days e. 21 days

b. Hyperthyroidism Hyperthyroidism is the most common endocrine disease in older cats. DM is also common in cats. Hyperadrenocorticism and hypothyroidism in cats are rare

What is the most common endocine disease in cats over 8 years old? a. Hyperadrenocorticism b. Hyperthyroidism c. Hypothyroidism d. Diabetes mellitus

a. Interruption of inhibitory neuron input from lumbar spinal cord With severe trauma between T2-T13, inhibitory pathways (esp. around L2-L4) are interrupted and cervical intumescence neurons (C6-T2) are "released," causing extensor hypertonia in the forelimbs. Think of Schiff-Sherrington syndrome when you see a combination of forelimb extensor rigidity and hindlimb flaccid paralysis, in an animal that has just had major spinal trauma, like being hit by a car

What is the neuroanatomic cause of extensor rigidity in the forelimbs of a cat with Schiff-Sherrington syndrome? a. Interruption of inhibitory neuron input from lumbar spinal cord b. Brachial plexus trauma c. Damage to thoracic spinocerebellar tracts in superficial white matter d. Polyradiculoneuritis of peripheral nerve sheaths at cervical intumescence e. Subdural white matter compression at C6-T3

d. Cardiomegaly The cardiac silhouette is markedly enlarged and rounded, occupying the entire width of the thorax on the DV projection. The pulmonary vasculature is normal in size, and there is no evidence of interstitial or alveolar pulmonary pattern. The pleural space and mediastinum are normal. DIfferential would include cardiomyopathy or pericardial effusion. Echocardiography revealed dilated cardiomyopathy.

What is the primary abnormality in these thoracic radiographs of a 5-year-old Siamese cat? a. Alveolar pattern b. Enlarged, tortuous pulmonary vasculature c. Mediastinal mass d. Cardiomegaly e. Interstitial pattern

b. Ileocolic artery The site for colonic resection is limited by tension on the ileocolic artery when trying to suture your new end of colon to the rectum. Sometimes the tension in too great and instead of a colocolic anastomosis, an ileocolic anastomosis must be performed. Essentially you are are trying to connect a section of ascending colon to the rectum. Now that can be pretty far Performing an ileocolic anastomosis is not ideal because you eliminate the ileocecal valve, and that may predispose the animal to bacterial overgrowth. The caudal mesenteric artery gives branches to the rectum and descending colon. The left colic artery also feeds the descending colon. The pudendal artery supplies the external genitalia. The ileocolic artery provides blood supply to the ascending and transverse colon.

When performing a subtotal colectomy on feline patient, what blood vessel limits the amount of colon that you are able to remove? a. Caudal mesenteric artery b. Ileocolic artery c. Left colic artery d. Pudendal artery

c. Mediastinum This cat has a cranial mediastinal mass. There is a small amount of pleural effusion causing widened fissure lines and retraction of the lung lobes. There is focal widening of the cranial mediastinum on the ventrodorsal projection, which is apparent as a soft tissue opacity flattening the margin of the cranial lung lobe on the lateral projection. The trachea is mildly elevated and narrowed in this area. This cat was diagnosed with lymphoma. Most cats with mediastinal or multicentric lymphoma are positive for feline leukemia virus (FeLV)

Where is the location of the main lesion in this cat? a. Lung parenchyma b. Pulmonary vessels c. Mediastinum d. Left ventricle e. Right atrium

d. Palpable nodule in the area of the thyroid gland A palpable thyroid nodule would be most supportive of hyperthyroidism in a cat out of all the answer choices listed. The other individual clinical findings alone are less specific to hyperthyroidism and are seen as signs in many other common diseases. Polyphagia can also be seen in diabetes mellitus, exocrine pancreatic insufficiency, interestinal parasitism, etc. Weight loss and low body condition can be due to cancer, diabetes mellitus, malnutrution, etc. Restless can be caused by pain, anxiety, pheochromocytoma, etc

Which clinical finding is most supportive of hyperthyroidism in cat? a. Polyphagia b. Low body condition score c. Weight loss d. Palpable nodule in the area of the thyroid gland e. Restlessness

a. Opioids NSAIDs and corticosteroids compromise the mucus-bicarbonate protection of the stomach. Renal disease and the corresponding uremia cause decreased mucosal blood flow and gastric hypersecretion. Liver disease causes gastric ulcers by decreasing mucosal blood flow secondary to portal hypertension and thrombosis. Liver failure is also associated with increased histamine and gastrin levels leading to gastric hypersecretion.

Which is not a common cause of gastric ulcers in dogs and cats? a. Opioids b. Corticosteroids c. Liver failure d. NSAIDs e. Renal failure

a. Acetaminophen

Which of the following agents produce methemoglobinemia and Heinz body anemia in cats? a. Acetaminophen b. Aspirin c. Calcium ethylenediamineteraacetic acid (EDTA) d. Ethyleneglycole e. Lead

e. Esophageal stricture Esophageal stricture is not a common post-op complication of thyroidectomy. Hypocalcemia can occur due to damage or excision of the parathyroid glands. Horner's syndrome occurs when the sympathetic trunk running through the neck is damaged. Laryngeal paralysis occurs with damage to the recurrent laryngeal nerve running through the neck. Hypothyroidism can occur secondary to removal of the affected thyroid gland or glands

Which of the following is not a common post-operative complication of thyroidectomy for hyperthyroidsim in cats? a. Hypothyroidism b. Horner's syndrome c. Hypocalcemia d. Laryngeal paralysis e. Esophageal stricture

d. Immune destruction of the thyroid gland causing a decrease of T4 into the normal range Immune destruction of the thyroid gland occurs in hypothyroid dog, but does not typically occur in cats. Euthyroid sick syndrome occurs when concurrent illness causes T4 to decrease from being high down into the normal range, or from the normal range to below normal. Fluctuations of T4 down into the normal range can occur early on in the disease. Occult hyperthyroidism occurs when the T4 is in the high normal range and the clinical signs are mild. In these pets, a free T4 by equilibrium dialysis can make the diagnosis of hyperthyroidism

Which of the following is not a common reason for a hyperthyroid cat to have a T4 value in the normal range? a. Fluctuation of T4 early on in the disease b. Mild disease in which there are subtle clinical signs and a T4 in the high normal range c. Concurrent disease causing euthyroid sick syndrome d. Immune destruction of the thyroid gland causing a decrease of T4 into the normal range

d. Thyroxine Thyroxine is the medical treatment for canine hypothyroidism. Methimazole is the medical treatment for feline hyperthyroidism. Radioactive iodine therapy and thyroidectomy are treatment options as well

Which of the following is not a treatment for feline hyperthyroidism? a. Thyroidectomy b. Methimazole c. Radioactive iodine therapy d. Thyroxine

d. Taurine deficiency

Which of the following is the most likely cause of dilated cardiomyopathy in an adult cat? a. Hyperthyroidism b. Neoplasia c. Pulmonary fibrosis d. Taurine deficiency e. Toxoplasmosis

a. Total T4 level This test is routinely available and reliable in the vast majority of hyperthyroid cats for diagnosing hyperthyroidism and monitoring therapy. Free T4 by equilibrium dialysis can be increased with diseases other than hyperthyroidism, so this test should always be paired with a total T4. The total T4 should be increased or in the high end of the normal range in conjunction with a high free T4, in order to make a diagnosis of hyperthyroidism. TSH levels are used to aid in the diagnosis of canine hypothyroidism (if they are increased), but they are not used in cats to diagnose hyperthyroidism

Which of the following is the screening test of choice for hyperthyroidism in the cat? a. Total T4 level b. Serum iodine levels c. Free T4 by equilibrium dialysis d. Thyroid-stimulating hormone levels

e. Diet change Diet changes such as a high fiber food or food designed to treat renal disease may be effective in restoring normocalcemia in the feline patient with idiopathic hypercalcemia. Prednisolone, fluid therapy, furosemide may be used in more severely affected patients and when dietary therapy is ineffective. Bisphosphonates may be utilized if increased osteoclastic activity is present.

Which one of the following choices is the best initial treatment in the patient with a moderate elevation of calcium caused by idiopathic feline hypercalcemia? a. Injectable fluids b. Prednisolone c. Furosemide d. Bisphosphonates e. Diet change

b. Progesterone Exogenous progesterone administration can lead to neoplastic transformation of mammary hyperplasia in intact or neutered male or female cats. Patients undergoing progesterone therapy that develop mammary hyperplasia should have the enlarged mammary gland(s) removed and submitted for historology in addition to the cessation of progesterone therapy.

Which one of the following hormones can induce neoplastic transformation of hypertrophied mammary tissue in cats? a. Oxytocin b. Progesterone c. Estrogen d. Testosterone e. Prolactin

b. Eosinophils The diagnosis of feline asthma is based on lower airway cytology with an increased number or percent of eosinophils. The other cell types, especially neutrophils, can be found, but are not suggestive of asthma. Bacterial infection should be ruled out with a culture of the BAL fluid. The treatment of feline asthma involves the use of corticostroids and bronchodilators, along with evaluation of a thorough history for any environmental triggers of the immune system

You are presented with a cat that is coughing at home, and you perform a bronchoalveolar lavage (BAL) or endotracheal wash. Which cell type, when present in BAL or endotracheal wash cytology, is suggestive of feline ansthma? a. Lymphocytes b. Eosinophils c. Basophils d. Neutrophils e. Monocytes

b. Urine culture All newly diagnosed diabetic patients should have their urine cultured. Diabetics are prone to getting urinary tract infections due to the chronic presence of glucose in their urine. Any underlying infection can lead to insulin resistance and make regulation of diabetes difficult to achieve. Blood pressure and ECG are good tests for all patients in general, but aren't necessarily test that are directly related to diabetes regulation A thyroid level should be checked in all older cats, for general health screening, but hyperthyroidism is not associated with diabetes mellitus. If a patient has diabetes which is difficult to control, concurrent hyperthyroidism should be ruled out. Fructosamine level is helpful in trying to determine if a cat actually has diabetes if their glucose level is elevated and also as a follow up to determine regulation. Cats that have markedly elevated glucose levels, gluco

You are starting insulin therapy for a newly diagnosed diabetic feline patient. Blood glucose is 520mg/dL (60-120mg/dL). Urinalysis shows 3+ glucose and is negative for ketones with a trace of protein. Which of the following tests should this patient and all diabetic patients receive upon initial diagnosis? a. Fructosamine b. Urine culture c. Blood pressure d. Electrocardiogram e. Thyroid panel

c. I-131 therapy The treatment of choice for hyperthyroidism in a case such as this would be radioactive iodine, or I-131 therapy. Molly would be a good candidate for this procedure since her renal values have remained stable while on methimazole. Tapazole is a brand name for methimazole, which is her current medication Thyroidectomy can be performed, but many times some thyroid tissue is left behind or there is ectopic thyroid tissue, which means the hypoerthyroidism continues or can recur at a later time. Also, this procedure carries more risk than does I-131 therapy. Imidocarb is an anti-protozoal medication and has no indication for treatment of hyperthyroidism Levothyroxine is a thyroid supplement used for treating hypothyroidism

You are treating Molly, an 8-year-old feline, for hyperthyroidism. She is currently on methimazole 2.5mg orally every 12 hours. Her labwork, including renal values, have been stable since diagnosis 2 months ago and her thyroid level is under control. The only concern is that Molly is experiencing extreme facial pruritis leading to excoriations. You believe this is due to a side effect of the medication. What other treatment option would be the best therapy for Molly? a. Thyroidectomy b. Imidocarb c. I-131 therapy d. Levothyroxine e. Tapazole

b. Continue 2U PZI BID and recheck BG curve in 2 months The cat is doing well clinically. The BG nadir is 101mg/dL, duration of insulin action is 9-10 hours, and the majority of your values are less than 250. All of these findings indicate good control of the diabetes, and no changes in treatment are necessary.

You are treating a 5-year-old male neutered cat that has diabetes mellitus. He is eating well at home and is no longer polyuric and polydipsic. You are treating him with 2U PZI insulin twice daily. His body weight is stable compared to his last visit. He ate a normal amount of food this morning and received his insulin before coming to see you. His blood glucose on presentation, 1 hours after receiving insulin, was 301mg/dL (normal is 60-125mf/dL). You evaluated a blood glucose curve with readings every 2 hours after the initial reading, and your results are: 273mg/dL, 233mg/dL, 130mg/dL, 101mg/dL, 248mg/dL. What would you recommend to this cat's owner a. Increase to 3U PZI BID and recheck BG curve in 1 week b. Continue 2U PZI BID and recheck BG curve in 2 months c. Decrease to 1U PZI BID and recheck BG curve in 1 week d. Switch toglargine insulin 2U BID, and recheck BG curve in 1 week e. Discontinue insulin, he is in diabetic remission

d. Ligation of thoracic duct and pericardectomy Surgery is the treatment of choice if medical therapy is failing. The best chance for resolution of a chylous effusion is ligating the thoracic duct and pericardectomy. Even with surgery, the effusion can still continue but this is the best chance for a cure. Somatostatin is a naturally occurring substance in gastric, pancreatic, and biliary secretions. In recent years, analogues of somatostatin have been used to successfully treat chylothorax in humans. The mechanism by which non-traumatic chylothorax may benefit from this treatment is unclear; however, resolution of pleural fluid (chyle and postoperative serosanguineous effusion) in both dogs and cats has occurred after administration of octreotide (somatostatin). It is extremely expensive and not as likely to cure the effusion as surgery would. Vitamin E and milk thistle supplements are sometimes used in case of l

You confirmed chyloys pleural effusion in your feline patient and referred to a specialist. The owner wants to discuss treatment options with you. The cat has had an extensive workups and no underlying cause has been found. Medical management has been tried for the last 3 months to no avail. The cat has been on a low fat diet and Rutin with intermittent thoracocentesis when needed. Which of the following treatments would be the best option and be most likely to resolve the effusion? a. Chemotherapy b. Vitamin E and milk thistle supplementation c. Place a chest tube to keep drained consistently for 1 week and this should resolve the fluid d. Ligation of thoracic duct and pericardectomy e. Somatostatin

a. Atelectasis Atelectasis or incomplete expansion of a lung due to loss of air from alveoli is a common complication of prolonged recumbency and inhalation anesthesia. Lobar consolidation differs from atelectasis in that it refers to filling of airyways with fluid. This occurs usually in inflammation. Pulmonary mineralization occurs from inflammation, infection, or neoplasia in the lung parenchyma. Pulmonary contusions are usually from trauma. The other main cause of atelectasis is decreased pulmonary surfactant in newborns or in ARDS or near frownings. Incidentally, another good answer to this question would be aspiration pneumonia

You have a cat that needs to stay on inhalation anesthesia and remain recumbent following a major surgery. What pulmonary complication is likely to occur from prolonged recumbency and anesthesia in this patient? a. Atelectasis b. Pulmonary mineralization c. Lobar consolidation d. Pulmonary contusions

b. Vaginal cytology showing mostly cornified epithelial cells The presence of cornified epithelial cells on a vaginal cytology swab helps to confirm that the cat is in estrus. The swab should be taken during the time the cat is exhibiting behaviour. In a queen under the influence of estrogen, you should see greater than 90% superficial cells If an LH level (lutenizing hormone) is low, an ovarian remnant is likely. If it is high, it does not rule it out Progesterone level is unlikely to be helpful because unless the cat ovulates, it will not rise Abdominal ultrasound is not usually helpful due to the small nature of a remnant and also because the remnant isn't always located at the ovary; it can also have a piece transplanted into other tissue such as the liver or omentum and can be very difficult to find. It is best to try and diagnose with cytology and then via exploratory laparotomu

You have been presented with a 3-year-old cat that was supposed to have been spayed at about 6 months of age. The current owner got her as a spayed cat, but every few months the cat is acting like she is in heat. Which of the following would help to confirm an ovarian remnant in this cat? a. Abdominal ultrasound b. Vaginal cytology showing mostly cornified epithelial cells c. Low serum progesterone level d. Elevated serum lutenizing hormone level e. Vaginal cytology showing bacteria and white blood cells with a low number of cornified epithelial cells

a. A low carbohydrate diet A low carbohydrate canned diet will help to improve glycemic control in a diabetic cat. It may even help to put the cat into diabetic remission for a period of time, where she will not require exogenous insulin. High fiber diets are helpful in the regulation of canine diabetes mellitus

You have been presented with a 9-year-old female spayed domestic shorhair cat with a history of polyuria, polydipsia, and weight loss. She has been eating a dry cat food that the owner purchased at the grocery store, labeled for adult cats. The cat's blood glucose is 400mg/dL (normal is 60-125mg/dL) and she has 3+ glucosuria (normal is negative for glucose in urine), diagnostic for diabetes mellitus. You start the cat on insulin. What should you recommended for her diet? a. A low carbohydrate diet b. A low fat diet c. A low protein diet d. A high fat diet e. A high fiber diet

e. Atipamezole Atipamezole (trade name: Antisedan), an alpha-2 antagonist, is the reversal agent for medetomidine (trade name: Dormitor). Medetomidine is an alpha-2 agonist. Xylazine would be another example of an alpha-2 agonist. Yohimbine is its reversal agent. Other alpha-2 agonists are clonidine, detomidine, dexmedetomidine, and romifidine. 2-pralidoxime is a reversal agent for cholinesterase inhibitors Flumazenil is a reversal agent for benzodiazepines Atropine is amuscarinic antagonist of acetylcholine and may be dangerous to use after administration of alpha-2 agonists. This is because alpha-2 agonists cause marked vasoconstriction and high afterload on the heart. Giving atropine and increasing the heart rate can place further stress on the heart. You do not want to see in motion peripheral vasoconstriction and compensatory bradycardia brought on by the alpha-2, then increase the heart rate against that h

You need to perform a CBC and take thoracic radiographs on a somewhat fractious cat. You sedate the cat with an intramuscular injection of medetomidine. While on the X-ray table, you become concerned that the cat is not doing well and you decide you want to reverse the effects of medetomidine. What should you give the cat? a. Atropine b. 2-pradidoxime c. Flumazenil d. Xylazine e. Atipamezole

b. This effusion is strongly suggestive of FIP FIP causes a classic straw or gold colored effusion with high protein content (about equal to serum protein) and moderate cell counts. An exudate would tend to have somewhat higher cell counts and lower protein. The protein content is not consistent with chyle or neoplasia

You perform thoracocentesis on a cat with pleural effusion. The fluid has a hazy gold color and a protein content of 7g/dL and 7,000 cells/uL. These cells are primarily non-degenerative neutrophils with lesser numbers of other white blood cells and mesothelial cells. What is the most appropriate interpretation of these findings? a. This is a chylous effusion b. This effusion is strongly suggestive of FIP c. This effusion is strongly suggestive of neoplasia d. This effusion is a classic exudate; the cat has pyothorax

c. Feline infectious peritonitis is not contagious and because her other cat died of FIP does not mean this cat will succumb to the disease Feline infectious peritonitis is not a contagious disease. It is a disease that is caused by a mutation of feline enteric coronavirus. It is unknown why in some patients this virus mutates and causes the FIP syndrome. It is most likely to occur in young or immunocompromised cats. Her other cat is not necessarily going to get FIP just from exposure. In fact, the majority of the cat population has been exposed to the feline enteric coronavirus. Becuase most cats in the general population have been exposed, it makes interpretation of coronavirus titers difficult. The titers can be elevated due to prior exposure and not from FIP. The titers can only be interpreted in lieu of clinical signs, blood results, etc. L-lysine is an anti-viral medication that may have some benefit for su

You see an 8-month-old kitten with the effusive form of feline infectious peritonitis and perform euthanasia. The kitten was having severe diarrhea around the house when it became ill. The owner has a 2-year-old cat at home and wants to know what this cat's prognosis is since it had been exposed to the sick kitten. Currently this cat is clinically healthy. What do you tell her? a. Perform a PCR on the cat's feces to see if the virus is being shed b. You recommend a coronavirus titer to determine it the cat is actively infectied c. Feline infectious peritonitis is not contagious and because her other cat died of FIP does not mean this cat will succumb to the disease d. Place the cat on L-ysine to prevent or supress any infection with FIP e. Her other cat may develop sysmptoms within the next two weeks because FIP is highly contagious

d. Have a housemate empty the litter box daily as a simple precauton to prevent infection as it takes 1-3 days for passed oocysts in the stool to sporulate into an infective form Toxoplasmosis gondii is a protozoal organism. The cat is the definitive host; the entire life cycle of the organism can be completed within this host. Most cats become infected when they consume an exposed rodent with bradyzoites encysted in their tissues. Only recently infected cats generally shed oocysts in their stool, and cats typically only shed these oocysts for 1-2 weeks. Most cats will only have one shedding episode in their lifetime. A IgM (not IgG) titer of 1:64 or greater suggests recent or active infection and that cat is at risk of shedding oocysts in their stools. Oocysts are not infective until they sporulate. This process takes > 24 hrs, so emptying the litter boxy daily is advised, preferably by someone who is not pregnant

Your client is pregnant and is worried about acquiring toxoplasmosis from her cat. What do you advise? a. Submit toxoplasmosis titers from the cat. A IgG titer of 1:64 or greater suggests recent or active infection that could pose a danger b. Toxoplasmosis titer should be performed on the owner by a human physician. A positive titer indicates antibodies to the organism that will prevent infection in the first trimester. A negative titer indicates she should remove her cats from her environment. c. Submite toxoplasmosis titers from the cat. A positive cat infected with toxoplasmosis can shed multiple times in its lifetime and pose a zoonotic risk d. Have a housemate empty the litter box daily as a simple precauton to prevent infection as it takes 1-3 days for passed oocysts in the stool to sporulate into an infective form


Related study sets

ECON 202 Final-Concept Questions

View Set

Econ ch 1 packet practice questions

View Set

Computer Literacy, Module 4 Computer Concepts Exam, TFS Module 2, IT 101, module 14 part 2, module 14

View Set

Chapter 16: Pregnancy at Risk: Conditions that Complicate Pregnancy Prep-U

View Set

Financial management study guide

View Set

Bio II: Chapters 18 - 21 (Test 1)

View Set